Contracts Semester 1

Pataasin ang iyong marka sa homework at exams ngayon gamit ang Quizwiz!

Assume that Carl's in good faith ordered the stated estimate of 200 pounds instead of the 1,000 in the pertinent month. Sammy's states that it is no longer going to deliver any scrap under the contract because, while it was making a few cents a pound when the contract started, it is now losing a few cents per pound under the deal because of increased energy costs. If Carl's sues Sammy's for failing to deliver, Carl's should: (A) Prevail, because 200 pounds is reasonable in light of course of dealing under the contract. (B) Prevail, because Sammy's is acting in bad faith by basing its decision on losing money. (C) Lose, because Sammy's has a valid commercial impracticability defense. (D) Lose, because the contract is unenforceable because it is illusory

A

Aunt Mary was a rich widow who loved her adult nephew Bill. However, Bill had come across tough times and had become addicted to cocaine. In order to help him put his life together, she invited him to her home and sat down with him in front of friends, family, clergy, and the local newspaper and told him, "Bill, if you forego cocaine for 5 years, I 48 will give you $100,000 to build a new life." Assume the jurisdiction follows the Restatement 2d. If Bill later seeks to enforce the promise after foregoing cocaine, will he prevail? (A) No, because the promise was not supported by consideration. (B) No, because it was oral. (C) Yes, because Bill's actions constitute a forbearance which is adequate consideration. (D) No, because the promise is between people who are related.

A

Becky, age 92, has only $25,000 to her name and is fearful that she will outlive her financial resources. On August 10, 2015, she describes her concern to her wealthy great-nephew Logan. On that same day, by signed writing, Logan makes her this promise: "When you exhaust the $25,000 that you now have, I will provide you, as a gift, any amount of money you request, up to a maximum of $50,000 per year, for the remainder of your life." In September 2015, Becky's grandson Thane asks Becky for $25,000. Unable to resist, and believing that she can turn to Logan for any money she may need, Becky gives Thane the $25,000 in her bank account — all that she has in the world. Becky then contacts Logan. Revealing what she has done, she asks him for $25,000. Logan responds, "I did not make my promise to you so that you could give your money away to Thane. I'm not going to keep the promise." To what extent does the doctrine of promissory estoppel require that Logan keep his promise of August 10, 2015? A.Not at all, because Becky did not reasonably or foreseeably rely on it B.To the extent of $25,000, because that is the extent to which Becky relied on it C.To the extent of $50,000, because that is the amount he promised to pay per year D.Fully, because Becky was reasonable in believing that Logan would honor his promise

A

Borrower owes $10,000 to Lender, but fails to pay it back. The debt consists of a loan with a $7,500 principal, and $2,500 interest on the principal. The statute of limitations has expired and thereafter Borrower tells Lender that he is sorry and promises, "I will pay you back $7,500 by January 31 of the following year" ("January 31 Promise"). Borrower fails to pay the agreed amount by January 31. In a jurisdiction that follows the Restatement 2d of Contracts, which of the following most accurately states Borrower's obligation to Lender? (A) Borrower enforceably owes Lender $7,500. (B) Borrower enforceably owes Lender $10,000 as the January 31 Promise effectively revived the debt. (C) Borrower owes Lender nothing because the January 31 Promise lacked consideration. (D) Borrower owes Lender nothing because the statute of limitations on the original debt expired.

A

Buyer and Seller enrolled in their employer's "Get Fit" program, which required all participants to purchase a treadmill to use in their office during lunchtime. Buyer and Seller were on different floors of the employer's office building. Buyer bought an inexpensive treadmill without any bells and whistles. Seller purchased a $2,000 state of the art treadmill with a television screen and calorie counter. After several months, Buyer was pleased to find that he could not get enough of the treadmill. He used it every chance he got. Seller, on the other hand, lost interest in the treadmill and dropped out of the "Get Fit" program. When Seller heard how much Buyer enjoyed the program, he wrote Buyer an e-mail 18 describing his treadmill and explaining he had lost interest in the fitness program. The e-mail provided, "If you pick up the treadmill at my office during business hours within the next three weeks, I will let you have it for $750." Buyer got busy at work and was unable to pick up the treadmill. Two weeks after Buyer received the offer, Seller's co-worker was at a meeting with Buyer and accurately told Buyer that Seller had already sold the treadmill to another employee. The co-worker was a good friend of Seller's and a trustworthy individual. Nevertheless, the next day, Buyer went to Seller's office to pick up the treadmill and presented Seller with a check for $750. Seller refused to sell the machine. The conversation between Buyer and Seller's co-worker (where the co-worker told Buyer that Seller had sold the treadmill to another employee) should be found by the court to be: (A) An indirect revocation of Seller's offer. (B) Of no legal effect because Seller's offer was irrevocable. (C) Of no legal effect because Seller's co-worker is a third party and cannot revoke on behalf of Seller. (D) A direct revocation of Seller's offer because no contract was formed when Buyer failed to send a letter back accepting the offer.

A

Buyer and Seller went out to dinner when Seller said "I will sell you this watch for $50." Buyer never responded and the two went their separate ways after dinner was finished. Three days after their dinner, Buyer called Seller and said: "I accept your offer; I will buy your watch for $50." This was the first communication between the two since the dinner. In a jurisdiction using common law presumptions regarding offer and acceptance rules, Buyer and Seller are: (A) Not in a contract, because Buyer did not accept at dinner. (B) Not in a contract, because the terms of the offer were too indefinite. (C) In a contract, because Buyer accepted Seller's offer to sell the watch for $50. (D) In a contract, unless Buyer changes his mind in which case he can void the contract since it was oral.

A

Danny Dude and his friends were having a wine tasting in Danny's wine cellar. Danny was having a good time and had become rather tipsy. He was slurring his words and stumbling a little. About an hour into the tasting, Danny was furious to find that he couldn't find his most precious bottle, a $2,500 bottle of 2000 Chateau Lafite Rothschild. Danny yells at the assembled crowd, "$250,000 to anyone who can produce a 2000 Chateau Lafite Rothschild." Everyone laughed at this ridiculous price, but Pearl took Danny seriously. Pearl slipped out of the tasting, immediately went to the nearest wine store, and bought a bottle of 2000 Chateau Lafite Rothschild for $2,500. Pearl brought the bottle back to the event before the wine tasting had finished. Danny's offer of $250,000 for the $2,500 bottle of wine is most likely: (A) Not an offer because a reasonable person would not believe Danny intended to offer $250,000 for a $2,500 bottle of wine. (B) Not an offer because the subject matter of the offer was missing. (C) An enforceable offer, but Danny can revoke the offer at any time if he changes his mind. (D) An enforceable offer, and Danny cannot revoke the offer because Pearl had materially relied on the offer by going to the store and purchasing the wine.

A

Ethel operates a business. To every employee she hires she makes what she calls her "four golden promises," shown in choices A-D below. Which of the golden promises subjects Ethel to a legal detriment? A.Ethel promises never to discharge an employee because he arrives a few minutes late to work. B.Ethel promises never to discharge an employee because of his religion. C.Ethel promises never to discharge an employee because she has joined a union. D.Ethel promises never to discharge an employee because he was born outside of the United States.

A

Eyon owns all of the stock in Eyon Inc. He has heard that Orlin, who owns all of the stock in Orlin Inc., might wish to sell it. In December, the two exchange these signed writings: December 1, 2019, Eyon: I may wish, within the next year, to purchase all of your stock in Orlin. I'll pay $15 million, but I need one year— until November 30, 2020—to make my decision. I am prepared to pay $100,000 tomorrow, December 2, in order to have from you this commitment: If, during this next year, I wish to make the purchase, you'll sell for that price. December 1, 2019, Orlin: Agreed. Please send the payment. Eyon sends Orlin $100,000. On December 2, Orlin receives it. Then, on May 1, 2020, Orlin contacts Eyon: May 1, 2020, Orlin: Not hearing from you in five months, I have decided not to sell you the stock. I revoke my offer and will return your $100,000. May 1, 2020, Eyon: No, no, no; that's not acceptable. As we agreed, I may still wish to purchase the stock and I will make my decision on or before November 30. QUESTION 8. Does Eyon retain the right, through November 30 to purchase the stock? A.Yes, because on December 1 he acquired a one-year option to do so B.Yes, because he proposed to pay the $100,000 within one day C.No, because corporate stock is not a good D.No, because Orlin stood ready to return Eyon's $100,000

A

For this question only, assume that BallCo sent SportCo its 2009 catalog but not the one it published in 2019. When BallCo insists that SportCo must buy the baseballs, SportCo agrees and stands ready to pay $288 per gross. BallCo demands $319, the price set forth in its 2019 catalog. All other facts are as originally set forth. Probably, BallCo is entitled to be paid A.$288, because that is the price it has repeatedly charged SportCo, and SportCo had no notice that the price had been changed. B.$288, because it is unreasonable for a seller suddenly to publish a higher price to a long-time buyer who has customarily and in good faith paid a lower one. C.$319, because under these circumstances a buyer should reasonably expect to pay according to a seller's published price list. D.nothing, because a buyer need not pay a price to which he has not agreed.

A

Harry owns a racehorse named "Thunder." It's a "mudder," meaning that it runs best on a wet, muddy track. Harry plans to enter Thunder in the upcoming National Horse Race. Jackson is a jockey, famous for his ability to ride mudders and famous, too, for his unwillingness to ride a mudder on a dry track. Harry and Jackson agree that if, on the day of the race, the track is muddy, Harry will ride Thunder. If it's dry, he won't. In exchange, Harry promises to pay Jackson $5,000, unconditionally, when the race is over. That means he promises to pay Jackson even if the track is dry, meaning that Jackson does not ride in the race. He agrees to pay Jackson even if, as things turn out, Jackson does nothing. On the day of the race the track is dry. Another jockey rides Thunder. Jackson does nothing. Because the track is dry, Thunder finishes in last place. When the race is over, Jackson demands that Harry pay him $5,000. Is Harry obliged to pay Jackson the $5,000 he demands? A.Yes, because Jackson's promise constituted consideration to Harry B.Yes, because Thunder did run the race C.No, because Jackson made only an aleatory promise and Harry made an unconditional one D.No, because as matters turned out, Harry derived no benefit from Jackson's promise.

A

In a face-to-face conversation, non-merchant Seller offers to sell his old smart phone for $100 to non-merchant Buyer. Buyer replies: "I'll pay you $75 for it." Seller says: "I cannot reduce the price, but my offer remains open to you until tomorrow." The next day Buyer tells Seller: "OK, I'll pay $100 for the phone." Are Buyer and Seller in an enforceable contract? (A) Yes, because Seller said the offer remained open. (B) Yes, because Seller made an irrevocable firm offer when he said the offer remained open. (C) No, because Buyer made a counteroffer and the power of acceptance was terminated. (D) No, because in a face-to-face conversation, the power of acceptance terminates when the conversation ends.

A

Landlord leased a commercial space to Tenant who wanted to use the space as a restaurant. The lease was for ten years and contained an "option to purchase" clause, allowing Tenant to purchase the property at the end of the lease term. The lease provided that the Tenant had to give notice of its intention to exercise the option six months before the lease ended, but contained no language about the purchase price or language indicating the parties agreed to agree on the price later. Tenant gave timely notice of its intention to exercise the option to purchase the building nine and a half years into the lease, but Landlord refused to sell, arguing there was no enforceable agreement as to the sale of the building because no price was set and no mechanism for determining the price was mentioned in the lease. In an action by Tenant against Landlord for breach of contract in a jurisdiction that follows the Restatement 2d, what is the likely result? (A) Judgment for Tenant, and the sale must take place at a reasonable price. (B) Judgment for Tenant, and the sale must take place at a price set by Tenant as winner of the action. (C) Judgment for Tenant, but Tenant can only seek monetary damages as the terms are too indefinite to order specific performance. (D) Judgment for Landlord, because there was no enforceable agreement as to the option due to indefiniteness.

A

Merchant Buyer ("Buyer") sends an order to Factory A, owned by Alvin, for 5,000 generic plastic paper clips. The offer could be validly accepted by beginning production of the clips. However, Factory A went out of business before the clips could be manufactured. As a courtesy to Buyer, Alvin forwarded the order to his friend Byron, the President of Factory B. Without notifying Buyer, Factory B competently fills the order and ships conforming clips to Buyer. Is there a contract between Buyer and Factory B? (A) No, because Buyer placed the order with Factory A. (B) No, because the clips were specially manufactured goods. (C) Yes, because Factory A validly transferred its right to accept when it transferred the order to Factory B. (D) Yes, because Factory B effectively accepted by sending the clips that Buyer ordered.

A

Owner, a resident of San Francisco, has a vacation home in Palm Springs that he only visits from November-March. Owner makes an offer in June to Painter to repaint the vacation home for $6,000, by September 1. In the offer, Owner let Painter know that Owner would not be at the vacation home until November. How can Painter accept? (A) Painter can accept by promise or performance, but if Painter accepts by performance, she must notify Owner or the payment duty of Owner is discharged. (B) Painter can accept by promise or performance, but does not need to notify Owner if she accepts by performance. (C) Painter must accept by promise. (D) Painter must accept by performance.

A

Paula Pound was out to dinner one night when she discovered her good friend, Darla Deed's wallet in the women's restroom. She took the wallet and after dinner started driving the wallet to Darla's home. On the way, Paula took a cell phone call from Adam Ant, who told Paula that Darla had just posted a $200 reward on her blog for the return of the wallet. Paula brings the wallet to Darla, and demands the $200 reward. Darla, however does not feel she should have to pay a friend who was going to return the wallet anyway, and refuses to give Paula the reward. Does Darla have to pay Paula the reward? (A) Yes, because Paula learned of the reward before she returned the wallet. (B) Yes, because Paula materially relied on the promise by driving the wallet to Darla's home. (C) No, because Paula did not know of the reward until after she began performance. (D) No, because Paula did not find out about the reward from Darla's blog, she found out about it from Adam.

A

Peter Pixie and Dean Dant were drinking together at Dan's house when Dean tells Peter he is dying to purchase his house. Peter, who was feeling tipsy, scoffed at the idea and said "You couldn't afford it you imbecile." Dean, who was unaware Peter was feeling the booze so much, responded, "Well, sure I could, I'll give you $500,000 for it." Peter shook his head and said, "No, make it $700,000," and Dean replied, "$600,000 and we have a deal." Peter said "Ok, $600,000 it is." Peter and Dean then wrote up a contract on a napkin that stated that Peter would sell his house to Dean for $600,000, payment was to be made by check on the following Wednesday, and the two even called Dean's wife into the room whereupon Peter, Dean and Dean's wife all signed the napkin contract. The following Wednesday, Dean went back to Peter with a cashier's check to tender payment. Peter yells, "You fool! I was only joking. I never intended to sell you my property. I was higher than a 747!" Assuming $600,000 was the fair market value for the home, in a suit between Peter and Dean, what is the likely result? (A) The two were in a contract if a reasonable person in Peter's position would believe they had entered into a contract. (B) The two were in a contract if Peter subjectively believed they were entering into a contract. (C) The two were not in a contract if Dean subjectively believed they were not entering into a contract. (D) The two were not in an enforceable contract because the napkin agreement does not satisfy the Statute of Frauds.

A

QUESTION 14. Now assume that when Nancy tests the sails, she finds them satisfactory. She sends Don this signed writing: "All of the sails function well, and I thank you for your work. However, five other sail makers tell me that $12,000 is out of line — that they would have done the same work for $8,000. For that reason, I will send a check for $8,000 as payment in full under our contract." Nancy does send an $8,000 check to Don, together with a letter that states, "I submit this check in accordance with my last writing, in which I noted that your $12,000 price was excessive." All other facts are as set forth in Question 11. Is Don entitled to receive the remaining $4,000 of the $12,000 contract price? A.Yes, because Nancy's debt to Don was fully liquidated B.Yes, because Don never expressly agreed to reduce his price by $4,000 after Nancy complained of it C.Yes, but only if Nancy failed to believe in good faith that $12,000 exceeded the prevailing market price D.No, because the parties achieved an accord and satisfaction

A

QUESTION 9. Which of writings 1-7 most likely constitutes an offer? A.5 B.5 and 6 C.4, 5, and 6 D.4, 5, 6, and 7 E.None of writings 1-7

A

RakeCo is a landscaper and CondoHome a residential condominium community. In each of the years 2009-2018, the two parties formed a written contract under which CondoHome paid RakeCo $90,000 and RakeCo attended to CondoHome's landscaping during the months of April through October. On February 1, 2019, CondoHome contacts RakeCo: (1) February 1, CondoHome (by signed writing): We have received no contract for this year. Please assure us that you'll take care of our landscaping this coming spring through fall. (2) February 4, RakeCo (by signed writing): Yes, of course we will. We have raised our fees by about 3 percent to all customers. In your case, our fee will be $93,000, not $90,000. None of our customers has objected to this modest increase, and we assume you will not object either. We'll be making all of our plans firm on April 1. You'll let us know, of course, if you have any comment. On February 7, CondoHome reads RakeCo's February 4 message and issues no response. On April 2, CondoHome advises RakeCo that for 2019 it has contracted with HoeCo, another landscaper, whose fee was $91,000, and that it will not accept RakeCo's services. RakeCo brings an action against CondoHome, alleging that the parties had formed a contract requiring that CondoHome retain RakeCo as its landscaper for April-October 2019 and pay its $93,000 fee. CondoHome contends that it never accepted RakeCo's February 1 offer, wherefore the parties formed no such contract. If judgment is for RakeCo, the reason will most likely be that A.RakeCo could sensibly understand that if CondoHome wanted not to accept RakeCo's offer, it would have said so before April 1. B.RakeCo did not impose the 3 percent fee increase on CondoHome alone, but rather imposed it on all of its customers. C.CondoHome should reasonably have anticipated that RakeCo might, after ten years, increase its fees. D.Before contracting with HoeCo, CondoHome owed RakeCo the opportunity to match HoeCo's price.

A

Randy Record also sued Camille for the $500 reward. Will Randy recover? (A) Yes, because Camille's attempted revocation was ineffective against Randy. (B) Yes, because Randy detrimentally relied on Camille's promise. (C) No, because Therese had already called about the record. (D) No, because Camille did not buy the vinyl that Randy told her about.

A

Seller offers to sell Buyer her antique record player for $80. Buyer replies, "Wouldn't you consider taking $50?" Seller says no. Buyer then says "OK, I'll take the record player for $80." No further communication existed between the parties. Has Buyer effectively and enforceably accepted Seller's offer? (A) Yes, because Buyer's response to the $80 offer never terminated his power of acceptance. (B) Yes, even though the power of acceptance briefly terminated when Buyer offered $50, Buyer then immediately accepted the original offer. (C) No, because Buyer had only a power, and not a right, to accept. (D) No, because Buyer made a counteroffer, which terminated the power of acceptance.

A

What are Professor Smith's obligations regarding Peter's widow? (A) He is not obligated to pay the widow anything. (B) He is obligated to pay $10,000, assuming her financial loss due to the death of her husband is at least equal to the $10,000 Smith promised to pay her. (C) He is obligated to pay $10,000 under the doctrine of promissory estoppel. (D) He is obligated to pay $10,000 because the promise was supported by consideration.

A

While negligently driving his father's uninsured car, Aaron crashed into a car driven by Brittany. Both Aaron and Brittany were injured. Aaron's father Carlos, reasonably but incorrectly believing that he would be liable as owner of the vehicle in the jurisdiction, told Brittany that he would "see that she was reimbursed for any losses" she incurred as a result of the accident. At all times relevant, assume Brittany had no thought of suing Carlos. In an action by Brittany against Carlos for lost wages while she recuperated from the accident, which of the following would be Carlos's best defense? (A) Lack of consideration. (B) Mistake of fact as to a basic assumption. (C) Statute of frauds. (D) Indefiniteness of Carlos's promise.

A

Alice and Barry are adults. They agree that for one year, Alice will not smoke and, in exchange, Barry will not drink. Does Alice suffer a legal detriment? A.Yes, because abstention from tobacco will benefit her health B.Yes, because Alice promises to refrain from exercising a legal right C.No, because apart from this agreement, Alice is free, legally, to smoke D.No, because Alice's abstention from tobacco will benefit her health

B

Alice and Barry are adults. They agree that for one year, Alice will not smoke and, in exchange, Barry will not drink. Does Barry enjoy a legal benefit? A.Yes, because abstention from alcohol will benefit his health B.Yes, because Alice promises to refrain from exercising a legal right C.No, because apart from this agreement, Alice is free, legally, to smoke D.No, because Alice's abstention from tobacco will not benefit Barry's health

B

All of the following took place in a jurisdiction that follows the Restatement 2d. Professor Smith looked out his front window one day when he heard some screeching tires. He immediately turned and saw his neighbor Peter hit by a car driven by Daniel. Dr. Jones, who was driving by, stopped and treated Peter and took him to the hospital. Despite Dr. Jones's rapid response and competent, expert treatment, Peter died from his wounds at the hospital a few days later. Professor Smith felt horrible about the whole situation because he had called Peter and asked him to come over because Professor Smith had some homemade cookies he wanted to give Peter. In order to cheer up Professor Smith, three of his students washed and groomed his prized poodle. The following day, to show his thanks, Smith promised the students he would pay them each $20. (The reasonable value of the wash and grooming was $150). Although Professor Smith was not legally responsible for Peter's accident, he felt responsible because Peter was coming to see him and because of that, he promised Peter's widow $10,000 a week after the crash. Who, if anyone, is obligated to pay Dr. Jones for the treatment of Peter? (A) Professor Smith. (B) Peter's Estate. (C) Both Peter's Estate and Professor Smith. (D) Neither Smith nor Peter's Estate is obligated to pay Dr. Jones.

B

BanaCo and StatCo deal in electrical equipment. In its catalog StatCo lists the prices for some of its products, but not for #4 conducting wire. On July 11, BanaCo sends StatCo this signed written message: "We wish immediately to have 4,000 feet of your #4 conducting wire. We do not see the item listed in your catalog, so we propose that price will be agreed on later. Please ship." On July 12, StatCo responds with this signed writing: "Thank you. We will ship." Have BanaCo and StatCo formed a contract? A.Yes, because where goods are bought and sold, price is not a material term B.Yes, because notwithstanding the open price term, their interaction reflects finality of agreement C.No, because two parties do not form a contract if they agree that a material term will be subject to future agreement D.No, because a seller who declines to publish its price shows an intention not to sell

B

Barry, a broke but aspiring chef heard that Francis French-Fry, a famous chef, was giving a free demonstration at a local cooking school. Barry went to the demonstration and was impressed with Francis's culinary skills. After the demonstration, Francis and his assistants went through the crowd to answer questions and let audience members know that Francis was giving cooking classes every Sunday morning for the next fourteen weeks for $550. For the next fourteen Sunday mornings, however, Barry showed up at the cooking school and participated in the baking lessons. Due to some administrative mix-up, Barry was not initially charged, but Francis is now seeking payment. What, if anything, will Barry be required to pay for the lessons? (A) Whatever is determined to be the reasonable value of the cooking lessons. (B) The full $550, under an implied-in-fact contract theory. (C) Nothing, because of the mix-up, it was reasonable for Barry to infer a gratuitous intent. (D) Nothing, because the price of the lessons was over $500 and there was no signed writing memorializing the contract.

B

Bill Benson wants to buy two adjacent farms, one owned by Sandra Salin, the other by Saul Simmons. But, Benson doesn't want to own either farm unless he owns both. On July 1, by signed writing wholly proper in form and content, Benson makes Sandra this offer: OFFER, made today, July 1, 2019: WHEREAS, from SANDRA SALIN ("Salin"), BILL BENSON ("Benson") wishes to acquire certain real property (the "Salin farm") [full legal description of the Salin farm] and wishes also to acquire certain adjacent real property owned by one Saul Simmons (the "Simmons farm") [full legal description of the Simmons farm] NOW, THEREFORE, Benson will, for $1.5 million, purchase the Salin farm only if, however, within 90 days from the date hereof, he succeeds in forming a contract with Saul Simmons, for the purchase of the Simmons farm for a price of $1.5 million or less. On that same day, July 1, 2019, Sandra reads the document. At its bottom, under Benson's signature, she writes "offer accepted," and signs her name. The parties sign a second copy of the same writing, each keeps one copy, and they separate. QUESTION 1. For this question, assume that one day later, on July 2, Sandra changes her mind; she doesn't want to sell her farm. She visits her attorney, shows him the signed writing, and asks, "Must I sell my farm to Benson?" With which of the following would the attorney respond correctly? A."If you revoke your offer before Benson begins to negotiate with Saul, then no, you are not obliged to sell your farm to Benson." B."If Benson succeeds in forming a contract for the purchase of Saul's farm, then yes, you will be obliged to sell your farm to Benson." C."No; Benson made no promise that he would attempt to purchase Saul's farm, and that renders your promise unenforceable." D."No; Benson has given you no consideration, which means your agreement is not a contract."

B

Dell Computers entered into an exclusive dealing agreement with Best Buy whereby Best Buy was granted the exclusive right to sell its computers nationwide in return for 60% of the sales price of any Dell computers sold, if any. The parties agreed to an initial shipment of 10,000 computers, with more computers to be shipped later, upon mutual agreement, once the initial shipment was sold. Which of the following best describes Best Buy's obligations to Dell Computers under the agreement? (A) Best Buy is not obligated to do anything to try to sell any Dell computers because Best Buy's obligation was illusory and not supported by consideration. (B) Best Buy is required to use "best efforts" in trying to sell Dell computers, but is not obligated to sell any computers. (C) Best Buy is required to sell 10,000 Dell computers, but is not obligated to order any additional computers. (D) Best Buy is required to sell all of the Dell computers delivered in the initial shipment, and is obligated to continue to receive shipments to sell in the future once those are sold.

B

Donee calls a St. Jude's telethon to contribute to the charity and pledges $500. However, when the charity later calls Donee back to inquire when to expect its payment, Donee tells them that she has changed her mind and that she will no longer be donating. In a jurisdiction controlled by the Restatement 2d of Contracts, in an action by St. Jude's seeking the pledge, who will prevail? (A) Donee, because the promise was a gift and not supported by consideration. (B) St. Jude's, because it is a charitable organization, even if St. Jude's did not rely on the pledge. (C) St. Jude's, but only if it can prove it relied on the pledge. (D) St. Jude's, but only if Donee is acting in bad faith.

B

Dr. Brown, the Head of Trauma Care at prestigious University Hospital, comes across an unconscious car accident victim in the road on her way home. She applies her considerable skill and stabilizes the severely injured man. However, the injuries were too severe, and the man dies in the hospital. 29 As one of the foremost trauma care specialists in the world, Dr. Brown routinely charges $2,000 per visit when she sees patients in her office. A normal trauma doctor in the area where she practices charges $400 per visit. If Dr. Brown sought payment from the injured victim's estate, which is the most accurate statement: (A) She would be entitled to $2,000. (B) She would be entitled to $400. (C) She would have been entitled to $2,000 had the victim lived, but is due nothing from his estate. (D) She would be entitled to nothing because she never entered into a contract with the victim.

B

Drugs, Inc. began a promotion for Sick-Me-Not, a medicine that is supposed to prevent users from catching any illness. Under the promotion, which was described in an ad that ran for three consecutive weeks on page 30 of Time Magazine, Drugs, Inc. told consumers it would refund the purchase price of Sick-Me-Not if purchasers used the medicine correctly for six months and still got sick. Piper Pepper purchased Sick-Me-Not and used it for six months, but still came down with bronchitis. She did not notify Drugs, Inc. that she was taking the medicine. Four months after Piper began using Sick-Me-Not, i.e., two months before the expiration of the six month period set forth in the ad, Drugs ran an ad for three consecutive weeks on page 30 in Time retracting the offer and saying it would no longer pay for anyone who became ill after using the product during the six month period she was using the product. Piper thereafter properly filled out the paperwork described in the first series of ads to receive her refund. However, Drugs, Inc. refuses to pay the refund amount. If Piper sues Drugs, Inc. for breach of contract, what is the most likely result? (A) Drugs, Inc. must pay Piper, unless Piper saw the ad retracting the offer before she completed performance. (B) Drugs, Inc. must pay Piper because she has begun performance and its attempted revocation was not effective against Piper. (C) Drugs, Inc. has no duty to pay Piper because Piper had a duty to notify Drugs, Inc. she was participating in the promotion within a reasonable time after she started, and she did not. (D) Drugs, Inc. has no duty to pay Piper because it made a general reward offer that was effectively revoked against Piper and any other use of the product who had not submitted the proper paperwork before the revocation ads appeared.

B

For this question, assume that Sandra remains happily committed to her contract. Instead, on July 2, Benson changes his mind. He doesn't want to buy either of the two properties, and tries not at all to contract for the purchase of the Simmons farm. Ninety days pass. On October 15, Benson advises Sandra that he has formed no contract to purchase Saul's farm and, therefore, will not purchase her farm either. Is Benson in breach of contract? A.Yes, because his offer to Sandra was irrevocable B.Yes, because he made no proper attempt to bring about the condition tied to his promise C.No, because his promise was subject to a condition that did not occur D.No, because the parties' agreement exacted no legal detriment from Sandra

B

Homeowner calls up Hardware Store ("Store") on the phone and asks the representative how much Store charges for Acme-brand, two-foot-high vinyl picket fencing. Store's representative said, "$2 per running foot." Homeowner said, "Great. I'll take 200 running feet and will be down this afternoon to pick it up." Assume no further communication or actions by the parties. Which is the most accurate description of the parties' status at this point in time? (A) Store has made an enforceable offer to Homeowner, but Homeowner has not effectively accepted the offer. (B) Store and Homeowner are not in a valid contract. ( C) Store and Homeowner are in a valid contract for 200 running feet of the fencing. (D) Store and Homeowner are in a unilateral option contract where Store is bound to sell 200 running feet of the Acme fencing at $2/running foot, but Homeowner can terminate the contract without consequence and need not pick up the fencing as promised.

B

In the town of Hampshire there live two women named Harriett Folger. One lives at 14 Chelton Lane. The townsfolk know her to be an industrial quality control engineer. She is also a talented seamstress but, as she is well aware, no one knows that. The other Harriett Folger, of 14 Chester Lane, is a seamstress, renowned for the high quality of her work. Farah Fuilan, also of Hampshire, writes and signs this letter: Dear Ms. Folger, As is well known, you are a gifted seamstress. I have a dress in need of repair. I'd like you to examine it and determine whether you can fix it. I'll pay you $100 to do just that much. I'd like to meet next Saturday — at 1:00 P.M. if that works for you. Are you agreeable? Sincerely yours, Farah Fuilan Confused as to which Harriett lives where, Farah addresses her letter to Harriett the engineer at 14 Chelton Lane. Harriett receives the letter, opens it, and writes back to Farah: "Yes, absolutely. I am agreeable. Let's meet this coming Saturday at 1:00 P.M., my home." Farah then learns of her error. She contacts Harriett the engineer and tells her, "By error, I sent my letter to you. I now know that I should have sent it to Harriett Folger the seamstress, at 14 ChesterLane." The engineer responds, "You do not know it, but I too am a capable seamstress. You sent the letter to me, I accepted your offer, and we have a contract." Farah asserts that she and Harriett (the engineer) did not form a contract. Her position is best supported by which fact? A.Farah honestly wished to contact the other Harriett and, under the circumstances, her error was reasonably understandable. B.Harriett knew that her abilities as a seamstress were unknown to the community. C.Harriett's principal professional activity is in the field of engineering. D.Farah cannot be bound to an agreement she did not intend to make.

B

On April 1, Fay sends to Mort this signed fax message: "Mort — April is upon us. I need your services — this Wednesday, please, at 11 A.M. I'll be ready with cash. Okay?" What additional circumstance, if proven, would most clearly mean that Fay's message constitutes an offer? A.Mort is a skilled landscaper, mechanic, and carpenter, and at various times in the past Fay has hired him to perform services related to those skills. B.Every April for the previous twelve years Mort has trimmed the trees in Fay's front yard, for which Fay has each time paid him $125. C.Fay does not know Mort, but has heard that he is a skilled landscaper. D.By signed writing, Mort responds to Fay: "Yes, I'll be there." E.By signed writing, Mort responds to Fay: "I'll be glad to help you, but my fee is now $35 per hour."

B

On January 4, Buyer, a nationwide electronics store, sent a Purchase Order to Seller, a television manufacturer that read, "Send 1,000 Model X-19 52-inch televisions, with delivery by May 15." Seller did not have enough televisions in stock, and so immediately began to manufacture televisions to fulfill the order. Seller has no further contact with Buyer until it shipped the televisions on May 8, with delivery guaranteed by May 15. Which of the following is correct? (A) Seller has not effectively accepted the order for televisions. (B) Buyer may treat the offer has having lapsed before acceptance. (C) Buyer would be bound to the contract once he received the televisions. (D) Buyer may treat the duties under the contract as discharged.

B

On July 1, Wharton and Xavier create a contract under which Xavier will pay Wharton $9,000 and Wharton will repair Xavier's large yacht, the Largesse. Wharton is to begin work on August 1 and on that day Xavier is to pay him $4,500. Wharton is to finish work on August 31, and Xavier is then to pay the remaining $4,500. Also on July 1, just after they have formed their contract, the parties converse: Xavier: I have another, smaller yacht. It needs that very same repair. For an additional $5,000 will you repair the smaller yacht just after you finish repairing this one? Wharton: I'll think it over and let you know within two weeks. Xavier: That's fine. Contact me by phone, fax, or email. QUESTION 5. One week later, on July 8, Xavier dies. As a result, Wharton loses I.his rights regarding the larger yacht. II.his rights regarding the smaller yacht. A.I only B.II only C.Both I and II D.Neither I nor II

B

On May 1, Lender lends Borrower $25,000, to be repaid on April 30 of the next year by ordinary personal check, together with interest of 5 percent, for a total of $26,250. On April 29, Borrower contacts Lender and explains that she cannot pay the full amount. Borrower asks whether Lender will accept $22,000 as a "settlement of the debt." Lender agrees. Borrower sends Lender a check for $22,000, together with a signed note indicating that the check is tendered "in full settlement of what was otherwise my total debt to you of $26,250." Lender deposits the check; three days later, Lender sends Borrower a signed writing: "Your check has cleared; you owe me nothing." Lender then decides that he wishes to recover the additional $4,250 originally owed him, and he brings an action against Borrower. The court issues judgment for Lender in the amount of $4,250. The court's reason for not enforcing the parties' settlement is most likely that I.the parties reached an accord, but the fact that Borrower owed Lender a liquidated debt of more than $22,000 means they failed to achieve satisfaction. II.in order that two parties effectively modify a contract between them, whether as an executory accord or in any other circumstance, each must provide the other with consideration. III.by first informing Lender, on April 29, of her inability to pay, Borrower gave Lender no fair and reasonable notice of her wish to settle the debt. IV.the parties were not eligible to achieve accord and satisfaction. A.I and IV B.II and IV C.III and IV D.I, II, II, and IV

B

On September 1, Roofer and Owner form a contract under which (a) Roofer will repair Owner's roof on or before October 1 and, in exchange, (b) Owner will pay him $1,500. For this question, assume that on September 12 Owner speaks to Roofer: "I'd like you also to replace my gutters. If you agree, I'll give you until October 3 to finish the whole job — roof and gutters. Will you do that without any increase in price?" Roofer replies, "Certainly. I'll finish the roof repair and the gutter replacement by October 3." After the September 12 conversation, Roofer is obliged by contract to A.repair the roof by October 1 and replace the gutters by October 3. B.repair the roof and replace the gutters, all by October 3. C.repair the roof by October 1 and nothing more. D.do nothing.

B

On Wednesday, Seller offers to sell Buyer her dining room table for $70. Buyer says "I don't know, will you give me a few days to think about it?" Seller responds, "Sure, how about until next Tuesday?" A few minutes later, though, Seller says, "I changed my mind, I want to keep the table and I am not going to sell it." Buyer then says, "I'll take it." Is there a contract between Buyer and Seller? (A) No, because Buyer rejected the offer when he said he was not sure and asked for more time. (B) No, because the power of acceptance was terminated when Seller said she was not going to sell the table. (C) Yes, but Buyer can avoid the contract because it was oral. (D) Yes, because Seller and Buyer entered into an irrevocable option contract when Seller told Buyer he could have until Tuesday, and Buyer validly accepted before Tuesday

B

Pam Poodle was opening a dog salon and needed someone to paint the salon. She wrote Bill Brush, a commercial painter, an e-mail stating "I'm opening up a dog salon next week and need someone to paint. If you come by and paint the shop, I'll pay you $2,500. Let me know, or just come by next week and start painting." Pam was out of town buying 9 supplies the next week, but Bill went to the salon and began painting. One week later, Pam saw Bill at the salon and told him she no longer needed him to paint the salon. Pam ordered Bill off her premises and has refused to pay him. Which of the following statements is most correct? (A) Pam could revoke her offer at any time before the job was completed. (B) A contract between Pam and Bill was formed when Bill began painting. (C) Bill could walk away before finishing, but Pam could not terminate him without incurring contractual liability. (D) Bill is entitled to the fair market value of the painting services he provided before Pam demanded he leave.

B

Pam saw an advertisement in the Sunday Times stating that Tar-Mart was hiring cashiers, and that all cashiers that are hired would start with, "a base salary of $60,000, with a 5% increase in pay guaranteed every six months the employee remains on the job." Pam applied and was hired for the job. She started at $60,000, but after eight months on the job, Tar-Mart had failed to give her a raise. Pam complained, and was told, truthfully, that there had been a good number of customer complaints about Pam's performance and therefore she had not performed well enough to merit a pay raise. Can Pam successfully sue Tar-Mart for breach of contract over the failure to increase her salary 5%? (A) Yes, because Tar-Mart made an effective offer when it advertised the employment opportunity, and Pam accepted the offer by applying for, and getting, the job. (B) Yes, because even though Tar-Mart did not make an offer by means of the advertisement, it set forth the terms of Pam's offer in the ad and is bound by the guarantee of a salary increase every six months so long as Pam is employed. (C) No, because advertisements are considered solicitations to make an offer, and not offers themselves. (D) No, so long as Tar-Mart was correct and in good faith stating that, under its standards, Pam had not performed well enough to earn the 5% pay raise

B

Paula, President of PowerCo, owns ten million gallons of diesel fuel and needs a refinery to process it during the coming autumn so she can sell it during the winter. Olivia owns and operates a diesel fuel refinery and Frank, her competitor, owns and operates one as well. On a Monday in August, Paul, Olivia, and Frank meet, each with a pad of note paper. They discuss Paula's need to have her ten million gallons of diesel fuel refined during the coming autumn. Olivia advises Paula that she has the capacity to do the job. Frank advises her that he, too, has that same capacity. On two separate sheets of paper from her pad, Paula writes, "PowerCo is prepared to pay $7 million now for a guarantee that our ten million gallons of grade-two diesel fuel oil will be refined on or before October 10 of this year." After she signs both writings Paula says, "Here, this is for the two of you." She then hands one to Frank and the other to Olivia. Frank and Olivia read the writing, and the three parties separate. On the next day, Tuesday, Frank personally delivers to Paula this written message, "I accept your offer of yesterday." On the same day, Olivia reaches Paula by telephone and says, "I accept the offer you made yesterday." Which of the following additional facts most likely means that Paula is obliged to purchase diesel fuel from Olivia and not from Frank? A.Frank did not sign the writing that he delivered to Paula. B.Olivia reached Paula by telephone before Frank arrived at Paula's office. C.When Frank wrote the words "I accept your offer of yesterday," he had lost Paula's writing and no longer had it in his possession. D.On Wednesday, Olivia sent by certified mail a signed writing in which she stated: "I hereby reaffirm acceptance, made by telephone on Tuesday, of Monday's offer."

B

QUESTION 10. Which of the following italicized modifications would most likely convert the indicated writing to an offer? A.Writing 1 is modified to read: "Shipment of Bortex Coal Kindler just received at reduced price. Prepared within next ten days to sell to you any quantity. Please accept promptly and advise us of the price you are willing to pay." B.Writing 2 is modified to read: "Re your offer of yesterday. We accept. We need 18 tons, $300 per ton. Please ship immediately." C.Writing 3 is modified to read: "We have 13 tons now available and will hold all until we hear from you. Price depends on amount ordered. Please respond soon." D.Writing 4 is modified to read: "Re Bortex—will certainly want all 13 tons delivered to our warehouse. Need price. Subject to price we have definite deal."

B

QUESTION 7. Assume now that both parties remain alive and well. On July 8, Xavier contacts Wharton and says, "I've changed my mind about both yachts. Repair neither one." As a result, Wharton loses I.his rights regarding the larger yacht. II.his rights regarding the smaller yacht. A.I B.II C.Both I and II D.Neither I nor II

B

QUESTION 9. Which of the following most accurately characterizes the transaction? A.Eyon offered to buy stock from Orlin. B.Orlin sold Eyon an option to purchase stock. C.Eyon was optionor and Orlin optionee. D.Eyon presented Orlin with an invitation to deal.

B

QUESTIONS 8-10. Beginning on October 1, FuelCo and HeatCo exchange signed writings: (1)October 1, FuelCo to HeatCo: Offer to Sell — Shipment of Bortex Coal Kindler just received at reduced price. Prepared within next ten days to sell to you any quantity. Please accept promptly. (2)October 2, HeatCo to FuelCo: Purchase Order — Re your offer of yesterday. We accept. What quantities are available, and at what prices? (3)October 3, FuelCo to HeatCo: Offer to Sell — Available: 13 tons. Price depends on amount ordered. What quantity shall we ship? Others are making orders. Supply is shrinking. Please respond soon. (4)October 4, HeatCo to FuelCo: Offer to Buy — Re Bortex — will want all 13 tons delivered to our warehouse. Need price. (5)October 5, FuelCo to HeatCo: Offer to Sell — Can deliver. Price: $800 per ton = $10,400. Please advise. (6)October 6, HeatCo to Fuelco: Purchase Order — Thank you. We expect delivery. (7)October 7, FuelCo to HeatCo: Purchase Confirmation — Okay. A.M. or P.M. delivery? QUESTION 8. Which of writings 1-7 are best characterized as invitations to deal? A.2, 3, and 4 B.1, 2, 3, and 4 C.1, 3, 5, and 7 D.1, 2, 3, 4, and 7 E.All of writings 1-7

B

Retailer sent Manufacturer the following e-mail: Please immediately send 500 widgets at the listed price of $5/widget. Widgets are urgently needed due to our big sale next week. /s/ Retailer Upon receipt of the e-mail, Manufacturer immediately sent Retailer the following response: We are out of stock of widgets. However, as an accommodation, we are sending you 500 of our semi-widgets at the listed price of $4/semi-widget. Most of our retailers have accepted the semi-widgets as a replacement for the widgets and we believe these will satisfy your customers. /s/ Manufacturer Manufacturer sent the semi-widgets the next day. Retailer never responded to the Manufacturer's e-mail, and the semi-widgets arrived in time for the sale. Retailer returned the semi-widgets rather than put them on sale. What is the best description of Retailer's rights arising out of this transaction? (A) By failing to respond to Manufacturer's e-mail, Retailer is obligated to accept the semi-widgets and must pay Manufacturer $4 for each one. (B) Retailer was not obligated to accept the semi-widgets; however, because it chose not to accept the semi-widgets, it has no right to sue Manufacturer for breach of contract. (C) Retailer is not obligated to accept the semi-widgets, and it is entitled to sue Manufacturer for breach of contract because Manufacturer sent non-conforming goods. (D) Retailer is not obligated to accept the semi-widgets; however, if it chooses not to accept them, it must allow Manufacturer a chance to cure by timely sending conforming widgets before Retailer is entitled to sue Manufacturer for breach of contract.

B

Seller offers to sell Buyer his vintage Corvette for $50,000. Buyer says, "Give me a week to think about it." Seller replies, "Sure, no problem." Buyer dies the next day. 19 Buyer's son, Buyer Jr., whom Buyer had told about the offer prior to his death, wrote Seller an e-mail the next day, correctly stating, "This is Buyer Jr., Dad died last night, but in his will he left the car to me. I am the administrator of his estate, so on behalf of his estate, I accept. Tell me where to wire the money." Seller has since decided to keep the car and refuses to sell it to Buyer's estate. If Buyer's estate sues Seller for breach of contract, the estate will: (A) Lose, because e-mail is not an effective mode of acceptance given the circumstances. (B) Lose, because Buyer Jr. did not have the power of acceptance even if he was the administrator of the estate. (C) Prevail, if Buyer Jr. can show that he materially relied on the offer. (D) Prevail, because he validly accepted the offer on behalf of the estate.

B

Seller put an ad in the paper saying he would sell his land for a minimum of $50,000. Buyer wrote him expressing interest in purchasing the land. Seller e-mailed Buyer telling her she needed to act fast if she was interested, because he had other offers for the property. The next communication from Buyer to Seller was an e-mail from Buyer stating that she had opened escrow with instructions to the escrow agent to pay Seller $50,000 upon receipt of good title to the property. Seller wrote back that he had already sold the property to another for $60,000. If Buyer sued Seller for breach of contract, which of the following is true? (A) Seller should prevail because Seller revoked his offer when he told Buyer she had to act fast. (B) Seller should prevail because they had not entered into a contract. (C) Buyer should prevail because Seller breached an enforceable contract to sell the property for $50,000. (D) Buyer should prevail because she materially relied on Seller's offer when she opened escrow.

B

The Restatement (Second) of Contracts rule providing that the death of the offeror terminated the offeree's power of acceptance is: (A) Consistent with the objective theory of contracts, but inconsistent with the subjective theory. (B) Consistent with the subjective theory, but inconsistent with the objective theory of contracts. (C) Consistent with both the objective and subjective theory of contracts. (D) Inconsistent with both the objective and subjective theory of contracts.

B

The law will LEAST likely regard as a contract which of the following exchanges? A.X will, today, convey to Y $100,000; Y will, today, convey to X the first U.S. postage stamp, issued in 1847, bearing a stated value of 5¢. B.X will, today, convey to Y 100 ordinary single-dollar bills; Y will, today, convey to X four ordinary twenty-dollar bills. C.X will, today, convey to Y one eight-ounce bottle of Coca-Cola for which X paid $1; Y will, today, convey to X one bottle of Pepsi-Cola for which Y paid 90¢. D.X will, today, lend Y $100,000; Y will, one year from today, repay the $100,000 together with an additional $5,000.

B

Tony's Trucking agreed to transport lumber from Larry's forest property in Oregon to Larry's lumber yard in Northern California. The price was fixed at $1,000 per truckload and the contract was for a four year term. Three years into the agreement, the price of gasoline unexpectedly and suddenly rose from $3.10 per gallon (the gas price at the start of the contract) to $4.50 per gallon. As a result of the increase, Tony is now losing money on each delivery. Tony asks Larry if he would be willing to raise the price from $1,000 to $1,250 per truckload for the remaining year of the contract which would allow Tony to stop losing money and break even. Larry agrees to this. In a jurisdiction which follows the Restatement 2d of Contracts, is Larry's promise to pay $1,250 per delivery enforceable? (A) No, because Tony is under a preexisting duty to deliver the lumber. (B) Yes, because the modification is fair and equitable in light of circumstances not anticipated when the contract was entered into. (C) No, because there was no consideration for the promise to modify their agreement. (D) Yes, because modifications never need consideration to be enforceable under the Restatement 2d of Contracts.

B

Buyer is a furniture store and a regular customer of Seller, who deals in vases. Once an order comes in, Seller selects the vases from inventory, tags them as Buyer's vases, packages them, and loads them onto a truck to be shipped to Buyer in trucks owned by Seller. On May 1, Buyer orders 100 vases with a Purchase Order that states, "Order for 100 green vases, Catalogue No. 223, at price per catalogue. Delivery by 6/1" Seller goes through the usual routine: picking the vases, tagging them and loading them onto the truck. Before the vases leave Seller's garage, which is the most accurate statement? (A) Buyer may revoke because Seller has not started to drive them to Buyer's place of business. (B) Seller may revoke without legal consequence, but Buyer cannot revoke. (C) Seller cannot terminate the deal, and Buyer cannot revoke without legal consequence. (D) Buyer may revoke without legal consequence because the goods have not yet arrived at Buyer's store.

C

David Bowie album. The David Bowie album she did not have was Space Oddity. She placed the following ad on Craigslist: $100 offered for information on the availability for purchase of a vinyl copy of David Bowie's Space Oddity album, in good condition. Contact Camille at (555) 555-5555. The ad was placed Friday, May 10 and was to run for a week, starting Sunday, May 12. Therese Tulip was another record collector, and independently knew that Camille was looking for the vinyl. On Sunday morning, she saw the record at a swap meet. Therese called Camille as soon as she left the swap meet and told her where she could find the record. Therese had not seen the Craigslist ad. Randy Record saw Camille's ad on Monday morning and called her right away to give her a tip on the availability of the record at a local record shop. Randy was the first person to call in response to the ad. Camille thanked Randy, but told him that she had already found the record she needed and did not want a second. Camille also truthfully told Randy that she had already contacted Craigslist at 10:00 a.m. that morning and cancelled the ad for the rest of the week, and therefore felt there was no longer a viable reward offer. A few days after telling Camille about the record, Therese found out about the ad and reward. She demanded payment from Camille who refused. If Therese sued for the reward will she prevail? (A) Yes, Camille must pay Therese because she bought the record based on her tip. (B) Yes, Camille must pay Therese because her revocation was ineffective against Therese. (C) No, Therese cannot collect because she did not know of the offer when she called Camille. (D) No, Therese cannot collect because Camille's ad was not an offer.

C

Dov owned retail store property at 199 Trumbull Avenue. Without naming a price, Portia expressed a tentative interest in buying it, and wished to know its market value. For that purpose, with Dov's permission, Portia hired and paid two professionals, both of whom reported to her and to Dov that it was worth approximately $450,000. Immediately thereafter the parties exchanged these signed writings: (1) Portia: As you know, I'm interested in your 199 Trumbull Avenue property, evaluated by two appraisers at $450,000. Will you sell it to me for that amount? (2) Dov: I'll take no less than $500,000. Talk to me when you're ready to pay that price. (3) Portia: Very well, $500,000 it is. Portia then tendered1 to Dov a $500,000 certified check and demanded that Dov convey the property. Dov refused and Portia sued, alleging breach of contract. Dov maintained that he breached no contract because he and Portia never formed one. Which of the following judicial statements plausibly constitutes a proper assessment of the conflict? I.Writing 1 was a mere invitation to deal. Writing 2, likewise, continued as a preliminary negotiation, manifesting Dov's willingness only to consider an offer of $500,000. Writing 3 was Portia's offer to buy for $500,000, which Dov did not accept. The parties formed no contract and Dov is not in breach. II.Knowing of Portia's interest in the property and allowing her to have it appraised, Dov implicitly offered to sell the property at any price reasonably close to the appraised value. With writing 1, Portia proclaimed her willingness to purchase for $450,000 and thereby accepted Dov's offer. The parties formed a contract and Dov is in breach. III.Writing 1 was a mere inquiry — an invitation to deal. Writing 2 was Dov's offer to sell for $500,000, and writing 3 was Portia's acceptance. The parties formed a contract and Dov is in breach. A.I B.I and II C.I and III D.II and III E.I, II, and III

C

For the five years 2015-2019, Nancy has lived with and cared for her elderly uncle Moses. At the beginning of year six, Moses's sister takes up the task. Nancy departs to begin the long awaited schooling she has thus far deferred. Grateful for Nancy's services, Moses creates a document entitled "Promissory Writing." He signs it and gives it to Nancy. Nancy reads it: IN CONSIDERATION OF and IN EXCHANGE FOR THE care, devotion, and affection she has shown and given me during these last five years I, Moses Shev, by this instrument to which I have set my hand and signature on this 26th day of February, 2019 do promise that should I live to the age of 90 (ninety) years I will, on my 90th (ninetieth) birthday give to my niece Nancy Shev the exact sum of $1 million. Two years later, Moses turns 90 but refuses to pay Nancy the $1 million he promised. Nancy brings an action against him for breach. Moses's attorney contends that Nancy gave no consideration for the promise, which fact, he says, renders it unenforceable. Nancy's attorney maintains that the years of service to which Moses expressly referred were Nancy's consideration for the promise. If the court rules for Moses, which of the following might it likely state as its reason? I.Nancy cannot demonstrate that her service had a monetary value of $1 million, wherefore Moses received no consideration for his promise. II.Nancy did not sign the promissory writing, wherefore Moses's promise does not belong to a contract and is, therefore, unenforceable. III.Moses did not bargain for Nancy's service, wherefore he gave his promise without consideration. IV.The promissory writing does not represent a bargain, wherefore Moses's promise is unenforceable. A.I and II B.II and III C.III and IV D.I, II, III, and IV

C

For the purposes of this question only, assume Alex began performance on May 4, and that Owner knew of this. If Owner's offer to Alex is considered an ambiguous offer, which of the following is true? (A) Alex may cease fixing the roof without breach, but Owner must hold his offer open for a reasonable period of time. (B) Owner can revoke his offer without breach, but Alex may not cease fixing the roof without breach. (C) Owner would breach the contract if he refused to go forward and revoked the offer, and Alex would breach the contract if she ceased fixing the roof. (D) They have no enforceable contract.

C

For the purposes of this question only, assume that the letter Betty sent by express mail was received on the morning of June 26, and Betty's first letter (the letter not sent by express mail) was received on the afternoon of June 26. If Sam sues Betty for breach of contract Sam will: (A) Lose, because the letter Betty sent by express mail did not have legal effect when it was sent. (B) Lose, because Betty's first letter had legal effect when it was sent, even though it was not received first. (C) Prevail, because Sam received the letter Betty sent by express mail before Sam received Betty's first letter. (D) Prevail, because the letter Betty sent by express mail had legal effect when it was sent.

C

For this question only, assume that BallCo did send SportCo a copy of its 2019 catalog, but that SportCo never examined it. When BallCo insists that SportCo must buy the baseballs, SportCo agrees and stands ready to pay $288 per gross. BallCo demands $319, the price set forth in its 2019 catalog. All other facts are as originally set forth. Probably, BallCo is entitled to be paid A.$288, because that is the price it has repeatedly charged SportCo, and SportCo had no actual knowledge of the price change. B.$288, because it is unreasonable suddenly to publish a higher price to a long-time buyer who has customarily and in good faith paid a lower one. C.$319, because under these circumstances a buyer should reasonably expect to pay according to a seller's published price list. D.nothing, because a buyer need not pay a price to which he has not agreed.

C

Hazel works for MedCo, and she happens to be an enthusiastic basketball fan. Nancy is her supervisor. Nancy approaches Hazel: "I have two tickets for this Friday night's game at the Garden. I'm going to give them to you tomorrow as an early birthday present." Hazel replies, "Thank you!" Is Nancy legally obliged to honor her promise? A.Yes, because she manifested a serious intention to honor it B.Yes, because she made it in consideration of Hazel's birthday C.No, because she received no consideration for it D.No, because she did not make it in writing

C

Jackson, a building contractor, contacts Leister, an electrical contractor. Jackson: I'm undertaking a construction project for Faith Hospital in Tuttontown. I need an electrical contractor. Interested? Leister: Yes, definitely, I accept. Jackson: Great. Can you come by my office tomorrow at 8:00 A.M., to iron out the details? Leister: Yes, I'll be there. At 7:00 A.M. the next day, Leister contacts Jackson announcing that she will neither keep the appointment nor serve as his electrical contractor. Is Leister in breach of contract? A.Yes, because Jackson made an offer and Leister accepted it B.Yes, because the parties manifested a mutual interest in forming a bargain C.No, because Jackson's first statement proposed no bargain D.No, because Leister timely notified Jackson of her altered decision

C

Lance, a landlord, owns two neighboring residential homes in Roseville. Simeon lives in one and Thad in the other, each as Lance's tenant. By their lease agreements with Lance, both tenants are obliged to "notify the landlord if the premises ever should suffer damage that demands immediate attention." Every year, Simeon winters in Florida while Thad remains in Roseville. Thad spends every summer in Maine while Simeon remains in Roseville. In the fall of 2018, Thad and Simeon form a contract under which •Thad will monitor Simeon's home during the winter and contact Lance if it should suffer damage that demands immediate attention; •Simeon will monitor Thad's home during the summer and contact Lance if it should suffer damage that demands immediate attention. After the parties form their contract, Lance, knowing nothing of it, approaches Thad. Lance: Your neighbor Simeon will be away for the winter. I make to you this proposal: You promise to watch his home and to contact me if it suffers any damage that needs immediate attention. In exchange, I'll promise to pay you $500 when Simeon returns home. Are we agreed? Thad: Yes, it's a deal. Winter comes and Simeon heads for Florida. All winter long, Thad dutifully monitors Simeon's home. Winter passes, the home is fine, and Simeon returns. Thereafter, Lance learns of the contract between Simeon and Thad. He refuses to pay Thad the $500 he promised. Is Lance in breach of contract? A.Yes, because in their agreement, both Lance and Thad suffered a legal detriment B.Yes, because when Thad and Lance reached their agreement, Lance knew nothing of the contract between Thad and Simeon C.No, because the agreement between Lance and Thad is not a contract D.No, because Simeon's lease imposed on him a preexisting duty to inform Lance of damage to his own home

C

Musician signed up for a monthly streaming music service like Spotify called Musicstream. Under their agreement, Musicstream automatically charged Musician's credit card $9.95 on the first of each month for that month's services. The agreement could be cancelled by Musician for any reason, but he had to give thirty days' notice to do so. Musician had checked out Spotify, which had a promotion going which allowed immediate cancellation with no extra payment, but Musician liked Musicstream's music library better, and signed up with that company. Musician was fired from his latest gig and so wanted to trim expenses. He e-mailed Musicstream on February 2 to cancel his account and to promise truthfully that he would no longer listen to any music from the service. Musicstream responded by telling him in a return e-mail that he would be charged for February, but that he would be charged for only one day in March. Musician wanted the charges stopped as of February 1. If Musician sued Musicstream in small claims court for a refund of the last $9.95 charge on his credit card for the period between February 2 and March 1, Musician should: (A) Prevail, because the terms of the Musicstream agreement are unconscionable. (B) Prevail, because charging of the credit card is an unenforceable acceptance by silence or inaction by Musician of Musicstream's offer for services for thirty days. (C) Lose, because Musician had given Musicstream reason to understand that assent for thirty days' worth of service may be manifested by silence or inaction. (D) Lose, because there is no consideration to support Musicstream's promise since Musician never listened to any music on the service during that last thirty day period.

C

On April 1, Elizabeth offered to sell Thomas the last puppy from the litter her dog had just birthed for $600. Thomas wanted some time to think about it, but Elizabeth knew her neighbor Amanda was dying to buy the puppy, and Elizabeth really needed the money. Because Elizabeth and Thomas had been best friends for a long time, Elizabeth told Thomas she would keep her offer open until April 8, in exchange for $5 from Thomas. Thomas gave Elizabeth the $5 and went home to think about the deal. Thereafter, Amanda gave Elizabeth $600 in cash for the puppy "just in case" Thomas did not want the puppy. After a long week of indecision about the responsibility of owning a puppy, Thomas woke up on the morning of April 8 and knew he had to have the puppy. He called Elizabeth immediately to tell her his decision, but she had not paid her phone bill and her line was cut off. Instead, Thomas wrote a check to Elizabeth for $600 and put it in a stamped envelope along with a note that he was accepting the offer to purchase the puppy. The envelope was properly stamped and addressed, and Thomas dropped the envelope in the mailbox at 10:00 p.m. on April 8. The note and check were not received by Elizabeth until April 10. When Elizabeth had not received the letter, or otherwise heard, from Thomas on April 8, she told Elizabeth the puppy was hers on April 9. 24. Who owns the puppy in a jurisdiction that follows the Restatement 2d of Contracts? (A) Thomas, because he had a valid option contract with Elizabeth and effectively accepted Elizabeth's offer to sell the puppy for $600 because, under the "mailbox rule," acceptances are effective upon dispatch. (B) Amanda, because oral option contracts are unenforceable. (C) Amanda, because Elizabeth sold the puppy to Amanda on April 9th before receiving Thomas's check. (D) Elizabeth, because the sale of goods priced $500 or more must be in writing or it is void.

C

On May 1, 2019, BuildCo and HotelCo formed a contract by which BuildCo was to build a hotel and complete its work by February 28, 2021. In exchange, HotelCo was to pay BuildCo $30 million. Knowing of HotelCo's plan to open a hotel, GasCo opened a gasoline station near the building site. GasCo speculated that the hotel would bring it business. In July 2020, GasCo learned that BuildCo was in financial difficulty and might fail to complete the hotel by February 28, 2021. On July 15, GasCo wrote to BuildCo: "We know, of course, that you are building a hotel for HotelCo, and we have heard that financial difficulties might prevent you from doing so. We make this proposal: If you will promise to complete the hotel by February 28, 2021 we will promise to pay you $2 million, beyond the moneys that HotelCo is to pay you." BuildCo wrote back: "Agreed. We will timely complete the hotel." On February 28, 2021, BuildCo completed the hotel. HotelCo has paid BuildCo $30 million, and BuildCo now demands $2 million from GasCo. Is GasCo obliged to pay BuildCo $2 million? A.Yes, because on July 15 GasCo and BuildCo exchanged signed, written promises B.Yes, because on July 15 GasCo and BuildCo formed an agreement that called for consideration from each C.No, because GasCo's promise was nudum pactum D.No, because HotelCo paid BuildCo $30 million

C

On May 1, Owner orally offered to pay Alex $15,000 if Alex would repair the roof on Owner's beach cottage. Alex requested some time to think it over. Owner told Alex she could have a couple of days to think about it, but that he had to know Alex's decision no later than May 4. On May 3, Alex sent a properly addressed e-mail to Owner, accepting the offer. She started working on the roof on May 4. If Owner's offer to Alex is considered an offer unambiguously looking for a unilateral contract, which of the following is true? (A) Alex had to notify Owner of her acceptance before she started repairing the roof, otherwise her acceptance was ineffective. (B) Alex's dispatch of the e-mail on May 3 was a valid acceptance of Owner's offer. (C) Alex may cease, without breach, if she has started to perform; but Owner must hold his offer open for a reasonable period of time. (D) Alex would be in breach if she walked off the job before completion.

C

On September 1, Roofer and Owner form a contract under which (a) Roofer will repair Owner's roof on or before October 1 and, in exchange, (b) Owner will pay him $1,500. QUESTION 5. For this question, assume that on September 12, Owner says to Roofer, "I'd like you also to replace my gutters by, let's say, October 3. Will you do that without any increase in price?" Roofer responds, "Certainly. I'll finish the roof repair by October 1, as agreed, and I'll replace the gutters by October 3." After the parties conduct that September 12 conversation, Roofer is obliged to A.repair the roof by October 1 and replace the gutters by October 3. B.repair the roof and replace the gutters, all by October 3. C.repair the roof by October 1 and nothing more. D.repair the roof by October 3 and nothing more.

C

Owner calls professional plumber to his home and describes the work he wants done. Plumber says, "I estimate the work will take four hours and I charge $60/ hour. So my estimate is $240." Owner says, "That sounds reasonable. Go ahead." Assuming no further communications between the parties, which is the most accurate statement? (A) The parties are in an enforceable bilateral contract for the plumbing work to be done for $240. (B) The parties are in an enforceable bilateral contract, but the price charged by the plumber could be more or less than $240, depending on how long the job actually takes, assuming she was acting in good faith both in making the estimate and in doing the work. (C) The parties are not in an enforceable contract. (D) The parties are in an enforceable unilateral option contract whereby the plumber is bound, but Owner may revoke any time before Plumber actually starts work.

C

Patty, a young college graduate, had saved up $5,000 while working on the weekends during her college years. She needed a car to go out for interviews to obtain a professional job and knew her uncle Donald had a car he was not using. She explained her situation to Donald who offered to sell it to her for the $5,000 she had saved to help her out with her job hunt, even though both Patty and Donald knew that similar cars were selling for no less than $8,000 on Autotrader.com and other car selling sites. She accepted on the spot and wrote him a check there and then, which Donald put in his wallet. However, the next day, when she went to pick up the keys and arrange for title to be transferred, Donald said he could no longer give her the car for that price and gave her back her check. In an action for breach of contract, who will prevail? (A) Donald, because the promise to sell was not supported by adequate consideration. (B) Donald, because until Patty picked up the keys and had the title transferred, the deal was not final and so he had the right to change his mind. (C) Patty, because there was a bargained-for exchange. (D) Patty, because of the moral obligation theory of consideration.

C

Paul Pack published an ad in his local newspaper offering "$500 to whoever provides information" that results in the recovery of Alice, his dog. Ann calls Paul on Tuesday and says she saw Alice at City Park near Paul's home. Dana calls Paul on Thursday and tells Paul she saw Alice in the same park. Paul went to City Park and found Alice on Friday afternoon. Paul paid Ann $500, but told Dana that he already had the information about Alice being in City Park before her call, so he did not believe he was obligated to pay her. Dana sues Paul for breach of contract. In the suit between Dana and Paul, Dana will likely: (A) Prevail, because Paul did not revoke the reward offer before Dana gave the information. (B) Prevail, because Dana knew about the reward offer before she provided the information and intended to accept it. (C) Lose, because Ann had already accepted the reward offer. (D) Lose, because ads are not offers, but rather solicitations for offers.

C

Priscilla and Sarah enter into an agreement by which Priscilla would build a custom patio cover for Sarah's new home. They agreed as to the price ($1,200) and the design, but they did not agree on a start or completion date, but rather they agreed to figure it out at a later date. Six months after they made the agreement, Priscilla had not started construction on the patio cover and there was no other communication between them. Sarah has now sued Priscilla for breach of contract. In a jurisdiction that follows common law rules, Sarah will likely: (A) Prevail, because six months is not a reasonable time to wait before beginning construction. (B) Prevail, if the contract was put in writing. (C) Lose, because the terms of the contract were too indefinite. (D) Lose, if the contract was not put in writing.

C

Sally Seller has an upscale clothing store on Rodeo Drive in Beverly Hills. She puts an advertisement in the LA Times that she is selling skinny jeans for $175, a bargain for this particular designer. Betty Buyer, a fashionista on a budget, was ecstatic to hear about the sale and went to the store that very same afternoon, as soon as she got off work. Betty pulled $175 cash from her purse and said, "I'll take the skinny jeans in a size 4, please." Sally had already sold out of Betty's size. She explained the situation to Betty, who was furious. Betty became even angrier when Sally refused to give Betty a "rain check," allowing Betty to purchase the jeans at the same price when her size came in. Betty sued Sally for breach of contract. In the suit against Sally Seller, Betty will likely: (A) Prevail, because Sally made a viable offer that Betty accepted by showing up at the store, with cash, to purchase the jeans in response to the ad and on the same day the ad ran in the paper. (B) Prevail, because while Sally was not in breach for running out of Betty's size, she was in breach for failing to give her a "rain check." (C) Lose, because Sally has not made a valid offer. (D) Lose, because Betty did not effectively tender performance.

C

Sarah Seller offers to sell Brianna Buyer her first year contracts book for $50, on the condition that Sarah's favorite singer, Angela Acapella, win "U.S.A. Icon," a popular singing competition. Brianna 20 accepts. Angela got knocked out of the competition early on. Brianna demands that Sarah still sell her the book, but Sarah refuses. Is Sarah contractually obligated to sell the book? (A) Yes, because Sarah made an offer to sell the book and Brianna accepted the offer. (B) Yes, because Sarah's condition was not related to the sale of the contracts book. (C) No, because Angela did not win "U.S.A. Icon." (D) No, because Sarah effectively revoked her offer.

C

Sealco manufactures seals, gaskets, and flanges. Bonaco manufactures a variety of plumbing products into which it incorporates jetline gaskets. On April 1, by signed writing, Sealco and Bonaco form a contract that includes these three paragraphs, 7, 8, and 9: 7. The parties acknowledge that Bonaco has and owns, now, a large supply of the jetline gaskets it uses in manufacturing various of its products, and cannot predict that during the coming period April 1 through March 31, that it will or will not have a need to acquire additional ones. Bonaco promises, covenants, and agrees that if it should have need of such additional units and should, therefore, purchase the same during the said period, it will purchase them from Sealco only. 8. In exchange for Bonaco's promise, covenant, and agreement as set forth in paragraph 7 immediately above, Sealco agrees to sell to Bonaco all such units, if any, as Bonaco may request, each for a price of $25 (twenty-five dollars) unless through no fault of its own, Sealco is itself unable to secure such units through its ordinary and customary channels and sources. Nothing herein shall prevent Sealco from selling such product during such period to other purchasers. 9. To each other, the parties make no promises or commitments beyond those set forth in Paragraphs 7 and 8 above. In June, from Sealco, Bonaco orders a large number of jetline gaskets, and stands ready to pay $25 per unit. Sealco advises Bonaco that it has decided to raise its price and will sell the units to Bonaco for $29 per unit. Bonaco complains that Sealco is in breach of its contract, to which Sealco responds, "We have no contract, because you have given us nothing, promised us nothing. With the agreement we signed, you promised not even to buy a single gasket. The agreement is unenforceable for lack of consideration on your part." Is Sealco right? A.Yes, because Bonaco made only a naked promise B.Yes, because Sealco made only a naked promise C.No, because Bonaco made a legitimate conditional promise D.No, because in June, Bonaco did order a large number of gaskets.

C

Shirley is a real estate broker. Tamara is a homeowner who wishes to sell her home. Tamara ("Seller") engages Shirley ("Broker") to represent her in the sale. By signed writing, the parties form an agreement with this substantive provision: Seller promises and agrees that for six months, Broker will have the exclusive right to list and show the home, and to offer it for sale on Seller's behalf. During that six-month period, therefore, Seller will not herself show the home to any prospective buyer and neither will she allow any other person to do so. The remainder of the writing carries various incidental terms, but recites no promises or commitments from Shirley. Tamara wishes to break her promise and allow another broker to show her home. Is she free to do so? A.Yes, because she made only a naked promise B.Yes, because her agreement with Shirley does not amount to a contract C.No, because she received consideration from Shirley D.No, because she made her promise in a signed writing

C

Uncle David, a successful businessman, made a promise to his 14-year-old nephew Paul in front of the 100 people present at David's 50th birthday party. Uncle David promised that if Paul refrained from playing football in high school and college, when Paul turned 21, David would give him $25,000. David was very fearful that Paul would suffer a concussion playing football and hoped that, with football out of the way, there would be nothing to stand in Paul's way to a successful business career and taking over David's company later in life. Paul refrained from playing the sport. Upon turning 21, he demanded the $25,000 promised, but David refused to pay. In an action to enforce the promise, who will prevail? (For purposes of this question, ignore any issues regarding the capacity of a teenager to contract and assume that Paul could enter into a contract in the relevant jurisdiction, and otherwise assume Restatement 2d of Contracts rules apply). (A) David, because David received no legal benefit from Paul's not playing football. (B) David, because the promise was oral. (C) Paul, because Paul's refraining from playing football was adequate consideration to enforce the promised payment. (D) David, because the promise is between people who are related.

C

Vlad telephones Esther and tells her, "I stand ready to clean your carpet for $125 on any Friday morning you choose." Esther repeats, "Any Friday morning?" Vlad answers, "Yes, my Fridays are clear." Esther responds, "I'll get back to you." "Very well," says Vlad, "I look forward to hearing from you." For two years neither party communicates with the other. Esther then telephones Vlad: "I accept your offer. Please come this Friday morning." Have these parties formed a contract? A.Yes, because Vlad made an offer and Esther accepted B.Yes, because Vlad expressly stated that "any Friday morning" would be appropriate C.No, because Esther failed to accept while Vlad's offer was operative D.No, because Esther rejected the offer by saying "I'll get back to you"

C

WesCo and JenCo exchange these signed writings: (1)WesCo: We need 25,000 units of Bantex 44. Please quote us a price. (2)Jenco (by signed writing): We have no Bantex 44 in stock. We probably can acquire the 25,000 units you request and deliver them within 20 days, but before we research that possibility, you should know that our current price is $8/unit. Interested? (3)WesCo: Your delivery terms are fine, but we are prepared to pay only $6/unit. Please procure product and ship for $6/unit. (4)JenCo: Would you consider paying $7/unit? (5)WesCo: No, thank you. We've changed our mind about the purchase; we'll do without the Bantex. (6)JenCo: We are able, after all, to order the 25,000 units, and we accept your offer to pay $6/unit. Will deliver. Thereafter JenCo contacts WesCo announcing that delivery is imminent. WesCo again advises JenCo that it no longer wants the goods and won't accept them. JenCo sues WesCo for breach. WesCo claims it formed no contract. Which of the following judicial conclusions properly supports a decision for WesCo? A.Neither party made an offer to the other. B.JenCo made an offer, but WesCo did not accept it. C.WesCo made an offer, but JenCo did not accept it. D.Each party made an offer, but neither made an acceptance.

C

At a family reunion, Pauline sat next to her grandmother Daniella. She told her that she was hoping to go to Paris the following summer after graduation. That spring, she received a card from her grandmother wishing her safe travels with a note stating: "Because of your trip, I will transfer $2,000 into your savings account when you return." Relying on the card and note, Pauline went to Paris and spent $1,000 of her savings on the trip. However, when she returned, Daniella refused to transfer the $2,000. 15. To what extent, if any, can Pauline enforce her grandmother's promise to pay her, in a jurisdiction governed by the Restatement 2d? (A) The promise is unenforceable because past consideration is insufficient to make a promise enforceable. (B) The promise is unenforceable because it was a gift promise. (C) The promise is enforceable, entitling Pauline to $2,000. (D) The promise is enforceable, but only for $1,000.

D

Benito is a graduate student. On September 15, 2019, he asks his friend Lannette for a $10,000 loan. Lannette: Why do you need the loan? Benito: The new semester has begun, and I'm $10,000 short on the tuition I owe. Classes have started and I've been attending them, but the bursar is demanding that I pay. Lannette: That's a lot of money, but I will never forget how you traveled across the country to visit me last year when I needed emotional support. Certainly I'll make the loan; you'll pay it back only if and when you're able. It will take me a week or two to get a hold of the money, but I'll put my commitment in a writing that you can show to the bursar. Maybe she'll leave you alone until the money comes through. Lannette then creates and signs this writing: September 15, 2019 To Whom It May Concern: In consideration of (i) my long-standing friendship with Benito Bourne, and (ii) the help he has given me and the sacrifice he has made for me in several specific instances, and (iii) his need to pay school tuition, I, the undersigned Lannette Lindstom as Lender, do hereby promise and commit myself to lend to the said Benito Bourne the sum of $10,000 at 1% annual interest, the loan to be made on or before October 1 of this year, 2019. REPAYMENT TERMS: The said Benito Bourne is to repay the loan and/or interest when and only when, if and only if, at any time in the future he feels himself ready, willing, and able to do so and otherwise not at all, neither loan principal nor interest. On October 1, Lannette tells Benito, "I'm sorry, but I've changed my mind about the loan. I don't think I can afford to part with the $10,000." Benito visits his lawyer who correctly advises him that Lannette's promise is unenforceable for lack of consideration from Benito. The lawyer is correct because I.notwithstanding her signed writing, Lannette made no promise to lend Benito $10,000. II.Benito promised nothing in return for Lannette's commitment. III.among the matters that Lannette cites as consideration for her promise, none was bargained for. IV.within Lannette's signed writing none of the terms provided at items (i), (ii), or (iii) truly describes consideration from Benito to Lannette. V.Benito did not sign the writing. A.I B.II and III C.I, II, III, and V D.II, III, and IV

D

Buyer ordered a truckload of a particular pesticide from Seller. Before Seller accepted the order, but after Buyer placed the order, the EPA outlawed both the sale and use of that particular pesticide. Seller thereafter accepts the order, believing that because the offer was placed before the EPA's actions, the order could be legally fulfilled. May Seller validly accept Buyer's order? (A) Yes, because Buyer placed the order before the EPA outlawed the pesticide. (B) Yes, so long as Seller's acceptance was sent within a reasonable time after the offer was placed. (C) No, because the time for acceptance has lapsed and the power of acceptance has terminated. (D) No, because the pesticide was outlawed before the acceptance was sent.

D

Contractor reaches an agreement with Supplier under which Supplier will provide a "truck-load" of lumber for a single-family home that Contractor is building. The parties agreed that the "truck-load" would cost $2000 and delivery was to take place at the jobsite on June 1. The lumber is to be Coastal Pine of excellent quality. Supplier believes that a "truck-load" is 1,000 pounds of lumber. Which of the following is an accurate statement? (A) The contract would be enforceable for 1,000 pounds of lumber if contractors in the business of building single-family homes understand a "truck-load" to be 1,000 pounds. (B) The contract would be enforceable for 1,000 pounds of lumber if Contractor has ordered seven "truck-loads" of lumber for past jobs from Supplier and each time Supplier delivered 1,000 pounds of lumber without objection from Contractor and were accepted by it. (C) The contract would be enforceable for 1,000 pounds of lumber if this were the third such "truck load" order for this specific job site under the current agreement between Contractor and Supplier and Supplier has delivered 1000 pounds of lumber in response to each of these orders without objection from Contractor and were accepted by it. (D) All of the above are accurate.

D

CupCo manufactured disposable cups and sold them to distributors. SupCo was a distributor of disposable cups, and on all the cups it sold, stamped its own name and imprint. On August 1, in its own truck, CupCo sent SupCo two thousand disposable coffee cups, in boxes, together with this message: We provide herewith two thousand of our new CoffeeFriend coffee cups. We assume you'll want to sell them to your own customers. If you keep them, we'll assume that you've decided to buy, and we'll bill you $125. If you do not wish to buy, please return to us within fifteen days, COD; we will pay the shipping charges. Which of the following would most likely mean that SupCo accepts CupCo's offer? A.SupCo keeps the cups in their unopened boxes for more than fifteen days. B.SupCo writes to CupCo: "We will keep the cups until you come to get them." C.SupCo writes to CupCo: "We very much like the cups." D.SupCo stamps its own name and imprint on the cups.

D

DevCo is a real estate developer and ConCo a builder. By signed writing, DevCo and ConCo form an agreement under which DevCo purports to hire ConCo to construct a housing development. The writing embodies 50 pages and 550 paragraphs. Paragraphs 1-20 identify the parties and define various terms. Paragraphs 21 and 22 provide: 21. ConCo will erect the housing development described herein, according to the timetables, schedules, specifications, and standards also provided herein. 22. DevCo will pay ConCo $25 million on each of the four dates herein named. Paragraphs 23-549 describe the timetables, schedules, specifications, and standards by which ConCo is to perform. They also describe the dates on which DevCo is to make its four $25 million payments. Paragraph 550 provides: The parties agree that any promise or commitment made by ConCo hereunder is conditioned on ConCo's determination that it wishes to honor it, which determination ConCo alone shall make if and as it wishes. In this agreement I.ConCo makes no promise. II.ConCo makes only an illusory promise. III.DevCo makes no promise. IV.DevCo makes a naked promise. A.I B.I and II C.I, II, and III D.I, II, and IV

D

Eunice, age 97, is fond of her younger neighbor Alison, age 26. On Wednesday, May 12, Eunice plans to move from her home in Washington State to New Mexico. On the evening before the move, she leaves Alison this voicemail message: "You've told me that you need a car. My car is nearly new, but I've decided to give up driving. In gratitude for our long friendship, I offer you my car. I'll sign the title certificate over to you and mark the box that reads 'transferred as gift.' I'll leave the certificate in your mailbox. I'm leaving for New Mexico tomorrow. Our next contact will probably be by mail." By leaving the voicemail message, Eunice A.did make an offer because she described her definite good faith intention to convey property to Alison. B.did make an offer because she expressed an explicit, unconditional plan, asking no response from Alison. C.did not make an offer because she did not require that Alison pay money for the car. D.did not make an offer because she did not call for a return performance from Alison. E.did not make an offer because she proposed not a commercial transaction but an informal arrangement between friends.

D

For the purposes of this question only, assume the following: when Adam tells Paula about Darla's reward, Paula responds, "I could never ask her to pay me for returning the wallet. That is what friends are for." Paula then returns the wallet and when Darla tries to pay her, Paula says "Don't worry about it, I won't take your money." Paula leaves without payment. However, on her way home Paula gets in a fender bender. She now has an unexpected payment to fix her car and could really use the reward money. Paula calls Darla and explains that she needs the reward money but Darla says that it is too late. If Paula sues Darla for the reward within the statute of limitations, what result? (A) Darla will lose, because she offered the reward and Darla accepted by returning the wallet. (B) Darla will lose, because Paula learned of the reward before she returned the wallet. (C) Darla will prevail, because Paula did not know of the reward at the time she decided to return the wallet. (D) Darla will prevail, because Paula did not intend to accept the reward offer at the time she returned the wallet.

D

LifeTime Inc. publishes a monthly magazine that derives revenues from advertising. As a promotional strategy the company offers various businesses the opportunity to advertise free of charge in three consecutive issues of its magazine. On April 1, a LifeTime representative telephones SportsWear: LifeTime: We're calling to tell you that free of charge in the next three issues of our magazine, we'll publish your standard full-page advertisement. SportsWear: We thank you. Please put your promise in a signed writing; we'd like to see it in black and white. LifeTime sends SportsWear this signed writing: Confirmation of Binding Contractual Promise This confirms that LifeTime will publish, free of charge, SportsWear's standard full-page advertisement in three consecutive issues of its monthly magazine. Thereafter, LifeTime's management decides against its free advertising program. There follows another conversation with SportsWear. LifeTime: Please know that we have changed our mind about the free advertising. We won't be publishing your ad. SportsWear: You made a signed written promise and we intend to hold you to it. Alleging breach of contract SportsWear sues LifeTime. LifeTime contends that it has no obligation to honor its promise. If the court decides against SportsWear, its reason will most likely be that I.SportsWear itself did not sign the writing entitled "Confirmation of Binding Contractual Promise." II.LifeTime's promise was not embodied within a contract. III.LifeTime withdrew its offer before SportsWear accepted it. IV.LifeTime's promise was mere nudum pactum. A.I only B.III only C.I and II only D.II and IV only

D

Martha secured a patent for a biomedical device she called "Gutmate." Believing that Deborah was a qualified manufacturer of biomedical devices, Martha sent Deborah the Gutmate plans and designs, together with a note: "I am prepared to hire you as manufacturer of Gutmate. I have little capital and propose, therefore, to convey to you a share of the patent ownership in exchange for your services. I invite your acceptance." Deborah responded, by writing: "Yes, we have a deal." Does Deborah's response constitute an acceptance? A.Yes, because Deborah expressed her assent, clearly and unequivocally B.Yes, because Martha explicitly invited Deborah to accept C.No, because Martha's writing omits to describe Deborah as a qualified manufacturer D.No, because Martha's writing proposes no definite bargain E.No, because it did not present the word "accept" or "acceptance"

D

Mr. and Mrs. Smith contract with Paul's landscaping service to "landscape our home in Orange County," twice a month for $150. Unknown to Paul, Mr. and Mrs. Smith have three homes in Orange County: one in Huntington Beach; one in Newport Beach, and one in Irvine. Paul shows up at the Huntington Beach home on a Saturday when the Smiths are there and they let Paul in the gate to perform the landscaping services. The grounds of the Huntington Beach home were the smallest of the Smiths' three homes, and the fair market value of his services at the Huntington Beach home was only $100. Now, Mr. and Mrs. Smith refuse to pay anything, claiming the contract was for the Irvine house. Their argument is the contract was unenforceable based on indefiniteness as to which residence was intended. In an action by Paul for payment, what result? (A) The contract is unenforceable because of the indefiniteness as to which residence was intended, but Paul can recover for the $100 fair market value of his services for the Huntington beach home on a quantum meruit basis. (B) The contract is unenforceable because it was not in writing, but Paul can recover for the $100 fair market value of his services for the Huntington Beach home on a quantum meruit basis. (C) The contract is enforceable because the contract is sufficiently definite regardless of the Smiths' conduct. (D) The contract is enforceable because Paul was allowed to landscape the Huntington Beach home by the Smiths, and Paul is entitled to $150.

D

On August 9, BuyCo and SellCo began communicating with these signed writings: August 9, BuyCo: We understand that you now own the original oil painting Nightbird, which is, of course, priceless. Nonetheless, we hereby offer to buy it, and we'd like to hear back from you on any other matters you deem significant, including price. Perhaps an appraiser's report would be helpful. We look forward to your response. August 11, SellCo: Thanks for your offer regarding our priceless painting, Nightbird. We accept, and we'll be in touch within ten days. Ten days later, on August 21, together with an appraiser's report that values it at $950,000, SellCo tendered delivery of the painting to BuyCo and demanded that BuyCo pay $950,000. BuyCo rejected the delivery and refused to pay anything, maintaining that "we never formed a final intention to buy." SellCo sues BuyCo for breach. Correctly deciding for Buyco, a court might make which of the following statements, whether or not it is a correct conception of law? I.The contract at issue is void for indefiniteness. II.The contract at issue is void for vagueness. III.The parties formed no contract because they agreed that the painting was "priceless." IV.The parties formed no contract because neither made an offer to the other. V.The parties formed no contract because BuyCo did not fully intend to purchase the painting. A.I B.I and II C.I, II, and III D.I, II, and IV E.I, II, IV, and V

D

On March 1, Dennis Storme telephones Mariah Wilson and leaves her this voicemail message: (1)March 1, Dennis (voicemail message): Hello, I'm Dennis Storme. You come highly recommended as a math tutor, and I'd like you to tutor my daughter Susan in trigonometry. Would you get back to me so that we can discuss the possibilities? (2)March 2, Mariah (voicemail message): Hello, Mr. Storme, this is Mariah Wilson. I would be happy to tutor Susan. I have only one opening at present—Saturday mornings from 10:30 A.M. to noon. My fee is $90 for the 90 minutes. I am prepared to commit to the next ten Saturdays. After that, we can consider whether Susan needs any additional tutoring. (3)March 3, Dennis (fax): Thank you for your return call. I cannot work with the hours you mention. Would you be able to tutor on Saturday evenings? (4)March 4, Mariah (voicemail message): No, Mr. Storme, I'm sorry that I cannot tutor Susan on Saturday evenings. Exactly as stated in my last message, I am available for the next ten Saturdays from 10:30 A.M. to noon and would be pleased to tutor Susan on those days at those hours. (5)March 5, Dennis (fax): Very well—we're agreed. Saturday mornings from 10:30 A.M. to noon; I'll find some way to see that we can do it. Legally, is Mariah free not to tutor Susan? A.Yes, because Dennis's March 5 message was too indefinite to create a contract B.Yes, because on March 3, Dennis lost his power to accept Mariah's March 2 offer C.No, because on March 3, Dennis plainly continued in his willingness to purchase Mariah's services D.No, because on March 5, Dennis effectively assented to the terms that Mariah had set forth on March 4

D

On May 1, Lender lends Borrower $25,000, to be repaid on April 30 of the next year by ordinary personal check, together with interest of 5 percent, for a total of $26,250. On April 29, Borrower contacts Lender and explains that she cannot pay the full amount. Assume now that on April 29, after advising Lender that she does not have the funds to pay her full debt of $26,250, Borrower says, "If you will accept $22,000 instead of $26,450, I'll pay you today, two days early. Furthermore, I'll pay you not by ordinary check but by certified check." Lender agrees. On that same day, Borrower engages a courier service that delivers to Lender a certified check in the amount of $22,000. Lender deposits the check, which clears three days later. A few days later, Lender undergoes a mood change. He sues Borrower for the additional $4,250 originally owed him. The court issues a judgment for Borrower. Its reason for enforcing the parties' settlement is most likely that I.Borrower paid two days before she was obliged to and did so in a manner that went beyond her contractual obligation. II.Lender received consideration in return for his willingness to accept less than the amount originally owed him. III.in exchange for Lender's willingness to accept less than the amount originally owed him, Borrower conferred on Lender a legal benefit. IV.in connection with the altered agreement, each party provided the other with consideration. A.I and IV B.II and IV C.III and IV D.I, II, II, and IV

D

On Monday, November 16, Homeowner told Painter she would give him $2,000 to paint the outside of her home by December 5, when she was hosting a party. Painter told Homeowner he would get back to her as soon as he looked up prices for the paint. Later that day, Painter called Homeowner and left a message on her answering machine that said, "This is Painter. I've checked the paint prices and I won't do the job for less than $3,500." A job Painter planned to work on fell through and Painter now needed the money. So on Friday, November 20. Painter called Homeowner and left another message, "This is Painter, again. Upon further consideration, I accept your offer to paint your house for $2,000. I'll do it this weekend." Homeowner played the second message just as she was going out of town for a long weekend away, and did not contact Painter. She had made no other arrangements for painting the house. That weekend, Painter went to Homeowner's home and painted the outside, without Homeowner's knowledge. When Homeowner returned from her vacation, Painter gave her a bill for $2,000. Homeowner refuses to pay the bill and Painter brings suit for breach of contract only. Who will likely prevail in the lawsuit? (A) Painter, because he accepted Homeowner's offer before Homeowner materially changed her position in reliance upon the first telephone message. (B) Painter, assuming the work he did was actually worth $2,000. (C) Painter, because he put Homeowner on notice that he was going to paint the home over the weekend. (D) Homeowner, because she did not accept Painter's offer to paint the home for $2,000

D

Owner owns a plot of undeveloped land in the state of Washegon. Owner went on an extended trip to Europe for several months. While Owner was gone, Contractor, who was having a slow month in terms of business, decided to build a home for Owner on the plot without talking to Owner. Contractor worked day and night for the several months Owner was gone and built a beautiful home. When Owner returned, Contractor told him, "If you use this house, you owe me $500,000 under contract law." Owner used the house, but was furious and refused to pay. Contractor sues Owner for breach of contract. 41. Contractor will: (A) Prevail, because in using the home when Contractor told him that if he did so, he would owe her $500,000, Owner is deemed to have accepted by silence. (B) Prevail, but will only recover the reasonable value of the home under quasi-contract principles. (C) Lose, if $500,000 is an unreasonable price for the home. (D) Lose, because Owner did not have reasonable opportunity to reject the services.

D

Paula and Dan Draper are a married couple with a new baby named Sarah. Dan hates giving Sarah baths. Just before Sarah's first birthday, Dan says to Paula, "I'll give you $1,500 if you bathe Sarah until she is two years old." 1. Assuming that a reasonable person would think that Dan was serious, is his promise to pay $1,500 enforceable? (A) Yes, because Dan is obtaining a legal benefit by the promise, and Paula is suffering a legal detriment from the promise. Thus, there is consideration to support the contract. (B) Yes, because a court would enforce Dan's promise on moral consideration grounds. (C) No, so long as a court finds that Paula would benefit from bathing Sarah. (D) No, unless a court finds the promise overcomes the normal presumption that governs such agreements.

D

QUESTIONS 13 & 14. By signed writing, Nancy and Don contract for the manufacture by Don of three new sails for Nancy's sailboat. The contract price is $12,000, and the contract specifies that the three sails will be suitable to function on Nancy's sailboat, with Nancy to have a reasonable opportunity to test their function before making payment to Don. Pursuant to the contract, Don inspects Nancy's boat and then begins to manufacture the sails: one a mainsail, one a jib sail, and one a mizzen sail. When Don announces completion of his work, he delivers the sails to Nancy. Before paying the agreed contract price, Nancy immediately tests their function on her sailboat. QUESTION 13. After conducting the test, Nancy concludes that the mizzen sail is, to some noticeable degree, not suitable for use on her vessel. She contacts Don by writing: "The mizzen sail is not quite right. It doesn't hang properly and doesn't trim quite right. I will send a check for $8,000 as payment in full under our contract." Nancy does then send an $8,000 check to Don, together with a letter: "I submit this check in accordance with my last writing, in which I described problems with the mizzen sail." Don deposits the check, it clears, and the $8,000 moves to Don's bank account. Thereafter, Don asks Nancy when she intends to pay the remaining $4,000 of the contract price. Nancy responds that she does not intend to pay it. She refers Don to the two writings she sent him after testing the sails, and to the $8,000 check she sent with the second one. Don responds that he never agreed to accept $8,000 as full payment and that he has taken the $8,000 only as a partial payment of the $12,000 owed to him. Nancy refuses to pay any additional amount. If Don brings an action against Nancy for the remaining $4,000 of the contract price, and Nancy proves at trial that she honestly believed the mizzen sail was unsuitable to her vessel, judgment should be for A.Don, because he received no consideration for the reduction in the contract price from $12,000 to $8,000 B.Don, because he never agreed to accept $8,000 as satisfaction of the $12,000 that Nancy owed under the contract C.Nancy, because Don's failure to perform one-third of the contract justifies her in withholding one-third of the price D.Nancy, because under these circumstances, Don's receipt of the $8,000 discharged her duty to pay the full $12,000 contract price

D

Rowdy Records is a music store in Indianapolis, Indiana. Rowdy's took out an advertisement in Monday's edition of the Indianapolis Star that read: "Freaky Friday Sale! Ten DVD's for only $1 each to the first customer who walks through our door this Friday!" Is the ad an enforceable offer? (A) No, because advertisements are solicitations for offers, not offers themselves. (B) Yes, under the UCC, but not under common law. (C) Yes under common law, but not under the UCC. (D) Yes, because all attributes of an enforceable offer are present.

D

Sally Seller and her friend Brian Buyer were at lunch one day when Sally told Brian, "I decided last night I am going to sell my motorcycle for $800." Brian, who thought this was a deal he couldn't refuse, responded "I accept, I will buy your motorcycle for $800." Assume $800 was a fair price for the motorcycle. Which of the following assertions is true at the time of Brian's statement? (A) Sally and Brian have entered into an enforceable contract and Sally is obligated to sell Brian her motorcycle for $800. (B) Sally and Brian have not entered into a contract. (C) Sally is an offeree with the power, but not the right, to enter into a binding contract with Brian to purchase the bike for $800. (D) Both (B) and (C) are correct.

D

Sam Seller tells Barry Buyer that he is considering selling his property. Sam invited Buyer to make an offer. Barry says: "How about $60,000?" to which Sam responds, "No, I wouldn't consider selling it for less than $75,000." Barry then says, "I accept, I'll pay you $75,000 for your property." There was no further communication between the two parties. Is there a contract between Sam and Barry? (A) Yes, because Buyer validly accepted Seller's enforceable offer to sell the property for $75,000. (B) Yes, because Seller's failure to respond to Buyer when he said "I accept, I'll pay you $75,000 for your property," constitutes an 5 implied acceptance on Seller's part, since Seller invited Buyer to make an offer on the property in the first place. (C) No, if the fair market value of the property was really $60,000. (D) No, because Seller did not make an enforceable offer, so Buyer did not have the power to accept.

D

Sam was a landscaper. Both Ashley and Betty owned homes and explained to Sam that they needed their front yards completely landscaped. On June 15, Sam mailed Ashley and Betty offers to landscape their yards. Sam offered to landscape Ashley's for $1,000, and offered to do Betty's yard for $1,500. Both offers stated that Sam's offer would be open until June 30. On the morning of June 20, Ashley mailed Sam a properly stamped and addressed letter stating, "I accept your offer." That afternoon, Ashley spoke with Zack, who said he'd do the job for $700. Ashley immediately mailed a letter to Sam by express mail that read, "Please disregard my other letter. I found someone else to landscape my lawn." On June 21, Sam received the letter Ashley sent by express mail. The next day, on June 22, Sam received Ashley's first letter. Sam demanded that Ashley permit her to do the work for $1,000, but Ashley refused. On June 24, Betty mailed a letter to Sam stating, "$1,500 is too much, I would agree to pay you $1,000 if you will agree to do the job." On June 25, Betty spoke to a few other landscapers, who said they would not do the job for less than $2,000. Realizing she could not get a better price, that afternoon Betty sent a letter by express mail to Sam stating, "Disregard my letter of 6/24. I accept your offer." Sam received both of Betty's letters on June 26. On June 27, Betty spoke to Thomas, another landscaper. Thomas said he could to the job for $1,000. Betty immediately called Sam and said (before Sam could say anything), "This is Betty, and the deal is off." Sam then demanded that Betty permit him to do the work for $1,500. Betty refused. If Sam sues Ashley for breach of contract, Sam will: (A) Lose, because Sam received the letter Ashley sent by express mail before he received Ashley's first letter. (B) Lose, because the acceptance in Ashley's first letter could be revoked up until the time Sam received it. (C) Prevail, because the letter Ashley sent by express mail did not have legal effect until Sam received it. (D) Prevail, because the offer in Ashley's first letter had legal effect when it was sent, even though Sam did not receive it first.

D

Same facts as above except: (1) assume there were no revocation ads, but (2) Alice Anderson used Sick-Me-Not for six months and 12 submitted the proper paperwork before Piper; and (3) Drugs, Inc. paid Alice's claim, but still refused to pay Piper. In response to Piper's suit against Drugs, Inc., what is the most likely result? (A) Piper will lose because she cannot effectively accept Drugs's offer because Alice accepted the general offer first. (B) Piper will lose because she did not notify Drugs that she was participating in the promotion. (C) Piper will prevail, but only under a promissory estoppel theory, because she relied on the offer. (D) Piper can validly accept, despite Alice accepting the offer first.

D

Sammy's Scrap Heap entered into a written contract with Carl's Car Parts under which Sammy's would supply, and Carl's would buy, all of the excess scrap metal that Carl's required from Sammy's. Sammy's obtained its supply of scrap from old cars that were donated or that it bought from the city impound. Carl's ordinarily needed 200 pounds of scrap every month, and that quantity was put in the contract as a good faith monthly estimate that Sammy's would be required to furnish. For six months, orders by Carl's varied between 175-225 pounds. However, one month Carl's received an extremely large order of car parts and ordered 1,000 pounds from Sammy's, which Carl's actually and in good faith needed, but Sammy's refused to supply that much. If Carl's sues Sammy's for failing to deliver the 1,000 pounds, Carl's will most likely: (A) Lose, because the contract lacked consideration. (B) Lose, because the offer to sell can be revoked at any time. (C) Prevail, as long as Sammy's is acting in good faith. (D) Lose, because of the stated estimate and previous purchases made by Carl's.

D

Seller offers to sell Buyer his ticket to a popular music festival for $200. Buyer says to Seller: "I plan to keep your offer under advisement, but while I am thinking about it, would you take $125?" Seller doesn't answer, but Buyer then says, "OK, I'll take it for $200." No further conversation occurred between Buyer and Seller. Is there a contract between Buyer and Seller? (A) No, because Buyer made a counteroffer which terminated his power of acceptance. (B) No, because the power of acceptance terminated at the end of the $125 conversation. (C) No, because Buyer has made an offer to purchase the ticket for $200, which Seller has not yet accepted. (D) Yes, because Buyer validly accepted the offer to buy the ticket for $200.

D

Shane sells books wholesale to retailers. Brenda is a retailer. On February 1, by signed writing, Brenda and Shane form a contract under which Shane will sell Brenda 500 copies of the novel Snowmaker, to be delivered on March 1, Brenda to pay a total purchase price of $750. On February 3, Brenda contacts Shane by signed writing: "Our contract calls for you to deliver 500 copies of Snowmaker on March 1. We'd like to have an additional twenty-five copies, 525 in all for the same $750 total, all to be delivered by February 15. Agreed?" Shane writes back, "Agreed." As of February 3, Shane's contractual obligation is to deliver A.500 copies of the book by March 1. B.500 copies of the book by February 15. C.525 copies of the book by March 1. D.525 copies of the book by February 15.

D

To say that Party 1 proposes (a) his own promise to do what the law on its own already requires of him, in exchange for (b) Party 2's promise to refrain from doing what the law otherwise allows — and that Party 2 assents to the proposal — is to say that I.Party 1 makes an offer but the parties form no contract. II.Each party makes a naked promise. A.I only B.II only C.Both I and II D.Neither I nor II

D

What are Professor Smith's obligations regarding the students? (A) He is legally obligated to pay the students $20 each as promised. (B) He is legally obligated to pay the students $150, which they must equitably split among them. (C) He is legally obligated to pay the students under the moral theory of consideration. (D) He is not legally obligated to pay the students anything.

D

While on a cruise, Alex, Brett's adult son, fell ill for several weeks. Another passenger on the ship, Cornelius, took care of Alex in his ill state. Alex did not have any additional money to pay Cornelius for his care, but Brett wrote to Cornelius expressing his thanks and promised to reimburse Cornelius for any expenses incurred in caring for Alex. Alex and Cornelius returned to their respective homes after the cruise, and Cornelius sent Brett a list of expenses from Alex's care. Brett refused to pay the expenses. In a suit by Cornelius to enforce Brett's promise, Cornelius will: (A) Prevail, because the promise was supported by bargained-for consideration. (B) Prevail, under a moral obligation theory of recovery. (C) Prevail, because the treatment of Alex is sufficient past consideration. (D) Lose, because the promise was not supported by adequate consideration.

D

Over the fourteen years, 2005 to 2018, Andrea has visited Frank's diner for breakfast about four times weekly between 6:30 A.M. and 11:45 A.M., each time ordering two soft-boiled eggs with toast, and each time paying the menu price for those items. Also over these fourteen years, Andrea has come to Frank's diner about three times weekly for lunch between 12:15 P.M. and 3:30 P.M., each time ordering two hardboiled eggs with toast and paying the menu price. At noon sharp on January 3, 2019 Andrea steps into Frank's diner and sits at a table. Which of the following additional facts would best justify the legal conclusion that on that day, Andrea has made an offer to Frank in which she proposes to buy soft-boiled eggs for the menu price? A.Unbeknownst to Frank, Andrea had breakfast today at 7:30 A.M. in another diner, which she also has frequented regularly for fourteen years. B.Unbeknownst to Frank or Andrea, Andrea's wristwatch is running one hour slow, and Andrea believes the time to be 11:00 A.M. C.Frank does not serve breakfast after 11:50 A.M. and begins serving lunch at noon. D.A moment after taking her seat, Andrea says, "Eggs and toast, please." E.A moment after taking her seat, Andrea says, "Another morning — the usual please."

E

Vortech Inc. supplied industrial chemicals to a large number of customers, including Hartco Inc. After years of doing business with Hartco, Vortech sent Hartco this document: DEFINITIVE OFFER TO SELL We have available a variety of alcohol- and ammonia-based cleaning products, many of which you've purchased in the past. We wish to sell some or all of the products in good quantities at favorable prices. Let us know, please, if you wish to purchase any now. Please describe specifically the product(s) you want to purchase and the exact quantities you wish to acquire. This is a definite offer, and we look forward to your acceptance. Did Vortech make Hartco an offer? A.Yes, because Vortech described its message as a "definite offer" B.Yes, because Vortech did not restrict Hartco to purchasing any particular product and left it free to choose among many C.No, because a seller cannot bind a prospective buyer without that buyer's assent D.No, because Vortech did not refer to cleaning chemicals by name E.No, because Vortech was nebulous as to the transaction it suggested

E

Alex invites Joan aboard his sailboat. Joan expresses concern for her safety. Among choices A-D, which of Alex's promises would NOT confer on Joan a legal benefit? A.Don't worry; I'll have on board all safety equipment required by federal and state regulation. B.Don't worry; I'll have on board more safety equipment than the law requires. C.Don't worry; I'll be more careful than any sailor ever has been. D.Don't worry; I'll see that you're perfectly happy, safe, and comfortable.

A

QUESTION 6. Assume, now, that neither party dies. Instead, on July 8, Xavier becomes mentally incompetent. As a result, Wharton loses I.his rights regarding the larger yacht. II.his rights regarding the smaller yacht. A.I only B.II only C.Both I and II D.Neither I nor II

B

Uncle told Nephew that he would give Nephew a valuable gold watch (worth $12,000) if Nephew would drive 400 miles to Uncle's house to pick it up. Uncle didn't like Nephew all that much, but Nephew was his only living relative and Uncle felt that his valuable watch should stay within the family. Nephew took two days off from work and drove the 400 miles to Uncle's house. When he got there, Uncle told Nephew that he had changed his mind and did not wish to part with the watch at this time. Does Nephew have a viable claim under any theory against Uncle? (A) No, because Uncle's promise lacked consideration. (B) Yes, if a court finds that justice requires enforcement. (C) Yes, because there was a bargained-for exchange. (D) No, because the promise was made to a family member.

B

Betty Buyer wishes to enclose her back yard with a fence. She reads reviews online and decides that Sam Carpenter is the best choice to construct the fence. On Monday, she leaves Sam a detailed message that includes the address, the exact type of fence she desires, the time for completing the project, a promise she will pay $600 for the job, and the fact that Sam can either call her back and accept or simply show up and begin work. On Tuesday, without speaking to Betty, Sam shows up at her house and begins to build the fence. When he's more than halfway through, however, Betty runs into her yard and yells "I revoke!" and orders Sam off her property. Betty: (A) Has validly revoked her offer because Sam never validly accepted it by completing performance. (B) Never made an effective offer. (C) Cannot revoke her offer because Sam has begun to perform. (D) May validly revoke, and Sam may also discontinue performing without consequence if he wishes to do so.

C

Jill has a large leak in her roof, but lacks the funds necessary to repair it. Her friend Jack voluntarily makes the repair. Three months later Jill learns that Jack is short of funds; he can't pay his rent. She contacts him and says, "I'll pay the $2,000 you owe for rent." When Jack protests that he cannot accept such a large sum, Jill responds, "You repaired my leaking roof when I couldn't afford to pay for it. I now promise you something in exchange. In consideration of the roof repairs you made for me, I will pay your August rent." Does Jack's roof repair serve as consideration for Jill's promise? I.Yes, because Jill made her promise expressly citing the roof repair as consideration II.Yes, because Jill manifested her serious wish to pay Jack's rent in exchange for the roof repair III.No, because the roof repair constitutes only past consideration IV.No, because Jill did not bargain for the roof repair V.No, because Jill did not record her promise in writing A.I and II B.I, II, and III C.III and IV D.IV and V

C

Assume Betty's first letter (the letter not sent by express mail) was received on the morning of June 26, and Betty's letter sent by express mail was received on the afternoon of June. If Sam sues Betty for breach of contract Sam will: (A) Lose, because Sam received Betty's first letter before Sam received the letter Betty sent by express mail. (B) Lose, because Betty's first letter had legal effect when it was sent. (C) Prevail, because Betty's first letter did not have legal effect until Sam received it. (D) Prevail, because the letter Betty sent by express mail had legal effect when it was sent, even though it was not received first.

A

Assume for this problem only that Plumber does the work described by Owner, but it reasonably and in good faith takes her 4 ½ hours to complete. She submits a bill to Owner of $270, constituting 4 ½ hours at $60/hour. It turns out Plumber is expensive for the area. Most plumbers would only charge $50/hour, although it would take them 4 ½ hours to do the work as well. As such, their bills to Owner would be $225, constituting 4 ½ hours at $50/hour. However, assume they, too, would have estimated that the job would only take four hours, meaning their estimate would have been 4 hours at $50/hour, or $200. How much does Owner owe Plumber? (A) $270 (4 ½ hours × $60). (B) $240 (4 hours × $60). (C) $225 (4 ½ hours × $50). (D) $200 (4 hours × $50).Exam Pro on Contracts,

A


Kaugnay na mga set ng pag-aaral

CompTIA A+ Exam 220-1001 - Network Protocols

View Set

Chapter 21: The Evolution of Populations

View Set

Chapter 5: Consciousness --> Expanding the Boundaries of Psychological inquiry (Quiz Questions)

View Set